You are on page 1of 82

Перейти до головного вмісту

• На головну
• Особистий кабінет
• Мої курси

sA











































Відкритий покажчик курсу
Відкрити ящик блоків

1. КРОК / KROK
2. Krok 2 Medicine (EN) 2023

Krok 2 Medicine (EN) 2023


Розпочато субота 17 червня 2023 21:09 PM
Стан Завершено
Завершено субота 17 червня 2023 21:10 PM
Витрачено часу 1 хв 3 сек
Балів 1,00/150,00
Оцінка 0,67 з 100,00 можливих
Питання 1
Відповіді не було
Макс. оцінка до 1,00

Відмітити питання
Текст питання
A 26-year-old woman complains of cramping abdominal pain, diarrhea with a significant
amount of mucus and blood, and fever of 37.5--38.0°С. Objectively, her skin and mucosa
are pale, the body type is asthenic. Palpation detects pain along the large intestine.
Colonofibroscopy reveals edematous wall of the rectum and sigmoid colon, erosions,
small ulcers, and mucus with blood in the lumen. What is the likely diagnosis in this
case?
Питання 1Відповідь
a.
Dysentery
b.
Chronic enteritis
c.
Crohn's disease
d.
Cancer of the large intestine
e.
Nonspecific ulcerative colitis
Коментар
Правильна відповідь: Nonspecific ulcerative colitis
Питання 2
Відповіді не було
Макс. оцінка до 1,00

Відмітити питання
Текст питання
A 64-year-old man fell to the floor, landing on his left side. Objectively, shortening of the
left limb and external rotation of the hip are observed. The patient is unable to perform
the straight leg raise test. During palpation and tapping on the heel, the patient feels
pain in the hip joint. What is the likely diagnosis in this case?
Питання 2Відповідь
a.
Contusion of the hip joint
b.
Greater trochanteric fracture
c.
Femoral neck fracture
d.
Fracture of the upper third of the femur
e.
Hip dislocation
Коментар
Правильна відповідь: Femoral neck fracture
Питання 3
Відповіді не було
Макс. оцінка до 1,00
Відмітити питання
Текст питання
A pregnant woman at 32 weeks of gestation with the risk of preterm labor undergoes a
treatment to prevent fetal respiratory distress syndrome. What medicine was she
prescribed?
Питання 3Відповідь
a.
Oxytocin
b.
Gynipral (hexoprenaline)
c.
Misoprostol
d.
Dexamethasone
e.
Progesterone
Коментар
Правильна відповідь: Dexamethasone
Питання 4
Відповіді не було
Макс. оцінка до 1,00

Відмітити питання
Текст питання
A 25-year-old woman had an abortion half a year ago. She complains of loss of appetite,
weakness, and arthralgia. Two weeks later, she developed dark urine and jaundice.
Against this background, her general condition continues to deteriorate. Viral hepatitis is
suspected. What marker of viral hepatitis is likely to be positive in the patient?
Питання 4Відповідь
a.
Anti-HBs
b.
Anti-HAV IgM
c.
Anti-HBc IgM
d.
Anti-HEV IgM
e.
Anti-CMV IgM
Коментар
Правильна відповідь: Anti-HBc IgM
Питання 5
Відповіді не було
Макс. оцінка до 1,00
Відмітити питання
Текст питання
A person fell from a ladder from the height of 2 m. An emergency medicine physician
suspects a vertebral compression fracture at the level of L1--L2. Objectively, the blood
pressure is 100/60 mm Hg, the pulse is 104/min. What aid must be provided to the
person at the scene of the accident?
Питання 5Відповідь
a.
Schneck's vertebral anesthesia, transportation into the inpatient department
b.
Analgesics intramuscularly, transportation in a sitting position
c.
Analgesia, transportation into a hospital on hard stretchers
d.
Transportation into the inpatient department in the position lying on the side
e.
Analgesia, antishock therapy, transportation into the inpatient department
Коментар
Правильна відповідь: Analgesia, transportation into a hospital on hard stretchers
Питання 6
Відповіді не було
Макс. оцінка до 1,00

Відмітити питання
Текст питання
A 55-year-old woman complains of diarrhea, irritability, and peeling and pigmentation of
the open areas of her body (neck, hands, and feet). Her history indicates that corn is one
of the main components in her diet. The woman rarely eats other vegetables and
legumes and does not eat meat or fish. What is the most likely diagnosis in this case?
Питання 6Відповідь
a.
Scurvy
b.
Psoriasis
c.
Pellagra
d.
Beriberi
e.
Biotin-responsive dermatosis
Коментар
Правильна відповідь: Pellagra
Питання 7
Відповіді не було
Макс. оцінка до 1,00
Відмітити питання
Текст питання
A 20-year-old woman is being treated for anemia (hemoglobin --- 72 g/L). A year and a
half ago, she had an involuntary miscarriage with a significant blood loss at 16 weeks of
pregnancy, after which the woman developed memory deterioration, fatigability, loss of
appetite, dry skin, brittle nails, and irregular menstrual cycles. Objectively, the following is
observed: blood pressure --- 80/55 mm Hg, pulse --- 54/min., height --- 168 cm, weight
--- 48 kg, genital hypoplasia. What medicine would be the optimal choice for this
patient?
Питання 7Відповідь
a.
Infliximab
b.
Human normal immunoglobulin
c.
Hydroxychloroquine
d.
Hydrocortisone
e.
Methotrexate
Коментар
Правильна відповідь: Hydrocortisone
Питання 8
Відповіді не було
Макс. оцінка до 1,00

Відмітити питання
Текст питання
A 65-year-old man with acute anterior myocardial infarction developed an asphyxia
attack. Examination detects diffuse cyanosis. His heart rate is 100/min. His blood
pressure is 120/100 mm Hg. Auscultation detects numerous heterogeneous moist
crackles in the lungs. What complication has developed in this patient?
Питання 8Відповідь
a.
Ventricular septal rupture
b.
Pulmonary embolism
c.
Pulmonary edema
d.
Hypertensive crisis
e.
Cardiogenic shock
Коментар
Правильна відповідь: Pulmonary edema
Питання 9
Відповіді не було
Макс. оцінка до 1,00

Відмітити питання
Текст питання
A 30-year-old woman came to a doctor with complaints of milk discharge from her
breasts and no menstruation for the last 5 months. She had one physiological childbirth
four years ago. The development of her breasts is normal. Bimanual examination detects
a reduced in size uterus and normal size of the ovaries. MRI detects no cerebral
pathology. Thyrotropin levels are within the normal range, prolactin levels in blood
serum are elevated. What is the most likely diagnosis in this case?
Питання 9Відповідь
a.
Pituitary adenoma
b.
Polycystic ovary syndrome
c.
Sheehan syndrome
d.
Hyperprolactinemia
e.
Hypothyroidism
Коментар
Правильна відповідь: Hyperprolactinemia
Питання 10
Відповіді не було
Макс. оцінка до 1,00

Відмітити питання
Текст питання
A 28-year-old woman came to a doctor with complaints of a sharp pain in her lower
abdomen and a short-term loss of consciousness at home. The last menstruation was 12
days ago. Vaginal examination shows that her uterus is of normal shape and painless; the
appendages on the left are slightly enlarged and painful to palpation; the posterior
fornix is overhanging, tense, and sharply painful. What is the provisional diagnosis in this
case?
Питання 10Відповідь
a.
Right-sided pyosalpinx
b.
Exacerbation of chronic adnexitis
c.
Apoplexy of the right ovary
d.
Ectopic pregnancy
e.
Pedicle torsion in a cyst
Коментар
Правильна відповідь: Apoplexy of the right ovary
Питання 11
Відповіді не було
Макс. оцінка до 1,00

Відмітити питання
Текст питання
A 19-year-old patient complains of a dry cough, muscle pain, and a fever of 39°C) A sore
throat and subfebrile body temperature were observed for the last week. Objectively, the
respiration is harsh. Complete blood count shows the following: leukocytes --- $10.0 ·
10^9$/L, leukocyte left shift, ESR --- 26 mm/hour. Chest X-ray shows an intensified
pulmonary pattern, low-intensity focal shadows in the lower segments of the right lung.
What medicines should be prescribed in this case?
Питання 11Відповідь
a.
Second or third generation cephalosporins
b.
Aminoglycosides
c.
Sulfanilamides
d.
Penicillin antibiotics
e.
Macrolides
Коментар
Правильна відповідь: Penicillin antibiotics
Питання 12
Відповіді не було
Макс. оцінка до 1,00

Відмітити питання
Текст питання
The patient's son has filed a lawsuit after his mother died of breast cancer. Due to the
fact that in the past she survived a stroke and had diabetes, aggressive cancer therapy
was contraindicated for her. She was suffering from severe pain, but doctors at the
outpatient clinic were unable to obtain even 1 mg of morphine for her treatment. What
kind of care should have been provided in this case by the primary care doctors to
relieve the patient's condition?
Питання 12Відповідь
a.
Palliative medical care
b.
Tertiary medical care
c.
Secondary medical care
d.
Primary medical care
e.
Emergency medical care
Коментар
Правильна відповідь: Palliative medical care
Питання 13
Відповіді не було
Макс. оцінка до 1,00

Відмітити питання
Текст питання
Forensic autopsy of the body of a 59-year-old man, who died suddenly at home without
signs of violent death, shows pink skin and mucosa, liquid bright-red blood, and bright-
red plethoric internal organs. Forensic toxicology blood testing detected 1.44 °/₀₀ of
ethanol in the blood and carboxyhemoglobin levels of 55 %. What is the cause of death
in this case?
Питання 13Відповідь
a.
Potassium cyanide poisoning
b.
Aniline poisoning
c.
Arsenic poisoning
d.
Carbon monoxide poisoning
e.
Alcohol poisoning
Коментар
Правильна відповідь: Carbon monoxide poisoning
Питання 14
Відповіді не було
Макс. оцінка до 1,00

Відмітити питання
Текст питання
After semolina was introduced into the diet, a 1-year-old child for 2 months has been
presenting with loss of appetite, irritability, loss of body mass, and copious and foul-
smelling stools. The skin is pale and dry, the hair is brittle. The abdomen is distended,
while the limbs are thin. Stool test shows high levels of fatty acids. What is the most
likely diagnosis in this case?
Питання 14Відповідь
a.
Functional diarrhea
b.
Irritable bowel syndrome
c.
Lactase deficiency
d.
Celiac disease
e.
Mucoviscidosis
Коментар
Правильна відповідь: Celiac disease
Питання 15
Відповіді не було
Макс. оцінка до 1,00

Відмітити питання
Текст питання
A girl with the weight of 3100 g was born at home from her mother's third pregnancy.
On the third day, the mother noticed small hemorrhages in the umbilical wound, as well
as "red dots" on the sclera of the child's eyes. The mother was not registered for prenatal
care checkups. She is breastfeeding her baby as needed. Physical examination detects
subconjunctival hemorrhages. Laboratory testing revealed that the hemoglobin levels
were 155 g/L, the mother's blood group was B(III) Rh-positive, while the child's blood
group was A(II) Rh-negative. What is the optimal tactics to prevent the progression of
this condition in the child?
Питання 15Відповідь
a.
Administration of vitamin K
b.
Administration of human antirhesus Rh0(D) immunoglobulin
c.
Transfusion of packed red blood cells
d.
Prescription of iron preparations to be taken orally
e.
Transition to formula-feeding with iron-fortified formulas
Коментар
Правильна відповідь: Administration of vitamin K
Питання 16
Відповіді не було
Макс. оцінка до 1,00

Відмітити питання
Текст питання
For the last 3 months, a 68-year-old woman has been suffering from pain attacks in the
heart that last for 10 minutes. The pain attacks occur in response to the slightest physical
exertion. She does not take nitroglycerin because of severe headache. She was
repeatedly treated for ischemic heart disease and had a myocardial infarction. Her blood
pressure periodically rises to 160/80 mm Hg. ECG shows cicatricial changes in the
posterior wall of the left ventricle. Auscultation reveals a systolic murmur over the aorta.
What is the most likely diagnosis in this case?
Питання 16Відповідь
a.
Aortic aneurysm with aortic dissection
b.
Unstable angina pectoris
c.
Recurrent myocardial infarction
d.
Stable exertional angina pectoris, FC II
e.
Stable exertional angina pectoris, FC IV
Коментар
Правильна відповідь: Stable exertional angina pectoris, FC IV
Питання 17
Відповіді не було
Макс. оцінка до 1,00

Відмітити питання
Текст питання
A 25-year-old parturient woman is hospitalized with contractions that have been
occurring for 12 hours already. The contractions last 25 seconds, while the intervals
between them last 3--4--7 minutes. The contractions are irregular and sharply painful,
with pain spreading upwards from the lower uterine segment. The baby is in the cephalic
presentation, the head is pressed to the entrance into the lesser pelvis. Uterine
hypertonus is observed. Internal obstetric examination shows that the cervix is smoothed
out and the opening of the external orifice of uterus is 3 cm. The amniotic sac is intact.
Make the provisional diagnosis:
Питання 17Відповідь
a.
Cervical dystocia
b.
Secondary weakness of the labor activity
c.
Discoordinated labor activity
d.
Primary weakness of the labor activity
e.
Physiological course of the labor
Коментар
Правильна відповідь: Discoordinated labor activity
Питання 18
Відповіді не було
Макс. оцінка до 1,00

Відмітити питання
Текст питання
For three weeks, a 29-year-old patient has been observing a painless ulcer on the skin of
the vermilion border of the lower lip. The last week, the ulcer became accompanied by
significant edema of the underlying and surrounding tissues. Examination shows an ulcer
up to 2.5--3 cm in diameter, with clear margins, the bottom that resembles "old lard" in
color, and an underlying cartilaginous infiltrate. What laboratory test must be prescribed
first in this case?
Питання 18Відповідь
a.
Smear for acantholytic cells
b.
Bacterial culture
c.
Eosinophil count
d.
Treponema pallidum test
e.
Skin scraping for mycosis
Коментар
Правильна відповідь: Treponema pallidum test
Питання 19
Відповіді не було
Макс. оцінка до 1,00

Відмітити питання
Текст питання
A baby with the gestation term of 30 weeks was born with the body weight of 1100.0 g.
Three hours after birth, frequent apneas appeared, "gasping" type of breathing is
observed, the heart rate is 98/min. The Silverman score is 9 points. Additional
oxygenation did not improve the child's condition. What must the doctor do next?
Питання 19Відповідь
a.
Tracheal intubation
b.
External cardiac massage
c.
Tactile stimulation
d.
Artificial pulmonary ventilation
e.
Positive pressure ventilation of the lungs
Коментар
Правильна відповідь: Positive pressure ventilation of the lungs
Питання 20
Відповіді не було
Макс. оцінка до 1,00

Відмітити питання
Текст питання
After the extraction of the second molar, the patient's body temperature increased. He
developed pharyngeal pain on the left, infiltration, hyperemia of the lower part of the
anterior palatine arch, and displacement of a tonsil towards the midline and upwards.
Regional lymph nodes are painful to palpation. The otolaryngologist diagnosed the
patient with paratonsillar abscess. What is the route of infection spread in this case?
Питання 20Відповідь
a.
Entry of a foreign body
b.
Tonsilogenic
c.
Odontogenic
d.
Lymphogenic
e.
Hematogenous
Коментар
Правильна відповідь: Odontogenic
Питання 21
Відповіді не було
Макс. оцінка до 1,00

Відмітити питання
Текст питання
A 7-year-old child complains of stomachache episodes that occur after mental stress,
cold drinks, or ice-cream. After clinical and instrumental examination the boy was
diagnosed with hypertensive biliary dyskinesia. What medicines should be prescribed
first for the treatment in this case?
Питання 21Відповідь
a.
Choleretics and cholekinetics
b.
Sedatives and cholekinetics
c.
Antispasmodics and choleretics
d.
Antibiotics
e.
Antioxidants
Коментар
Правильна відповідь: Antispasmodics and choleretics
Питання 22
Відповіді не було
Макс. оцінка до 1,00

Відмітити питання
Текст питання
A 32-year-old patient looks at the pattern on the wallpaper and sees the lines begin to
move and form silhouettes of fantastic animals. Instead of a lighting fixture hanging
from the ceiling, he sees a giant octopus. What psychopathological symptom is it?
Питання 22Відповідь
a.
Pareidolic illusions
b.
Derealization
c.
Pseudohallucinations
d.
Visual hallucinations
e.
Functional hallucinations
Коментар
Правильна відповідь: Pareidolic illusions
Питання 23
Відповіді не було
Макс. оцінка до 1,00

Відмітити питання
Текст питання
A 16-year-old boy developed dizziness. Objectively, his heart rate is 35/min., his blood
pressure is 85/45 mm Hg, the heart borders are not enlarged, the heart sounds are loud
and clear. ECG shows P waves disconnected from QRS complexes, dissociation and
different rhythm of atria and ventricles are accompanied by varying location of P wave in
relation to QRST complex. This presentation is most characteristic of the following
disease:
Питання 23Відповідь
a.
Complete atrioventricular block (III degree)
b.
Sinus bradycardia
c.
Atrioventricular block (II degree)
d.
Atrioventricular dissociation
e.
Extrasystole
Коментар
Правильна відповідь: Complete atrioventricular block (III degree)
Питання 24
Відповіді не було
Макс. оцінка до 1,00

Відмітити питання
Текст питання
A 26-year-old pregnant woman was hospitalized into the intensive care unit at 36 weeks
of her pregnancy with complaints of an intense headache in the frontal region. Physical
examination detected the following: blood pressure --- 170/90 mm Hg, pulse ---
85/min., respiratory rate --- 15/min., body temperature --- 36.9°C, edema of the limbs,
fetal heart rate --- 159/min. During the examination, the woman developed an attack of
generalized tonic-clonic seizures. What drug must be administered first in this case?
Питання 24Відповідь
a.
Diazepam
b.
Magnesium sulfate
c.
Lamotrigine
d.
Sodium valproate
e.
Phenytoin
Коментар
Правильна відповідь: Magnesium sulfate
Питання 25
Відповіді не було
Макс. оцінка до 1,00

Відмітити питання
Текст питання
A 45-year-old patient was referred for a consultation with a psychiatrist due to
complaints of abdominal pain and discomfort that occur in emotionally straining
situations. Objectively, no changes of the gastrointestinal tract were detected. The
complaints first arose over 10 years ago against the background of a severe alcohol
poisoning. The patient has been repeatedly visiting gastroenterologists, who were
unable to find any gastrointestinal pathology in the patient. The prescribed therapy was
ineffective. What condition is most likely in the patient?
Питання 25Відповідь
a.
Chronic alcoholism
b.
Somatoform autonomic dysfunction
c.
Organic brain disorder
d.
Depressive disorder
e.
Functional dyspepsia
Коментар
Правильна відповідь: Somatoform autonomic dysfunction
Питання 26
Відповіді не було
Макс. оцінка до 1,00

Відмітити питання
Текст питання
Preventive examination of the population of one of the Ukrainian cities detected spotted
tooth enamel and generalized osteosclerosis with calcification of the intervertebral
ligaments in 25 % of the residents. What is the most likely cause of these symptoms?
Питання 26Відповідь
a.
Insufficient fluorine levels in animal products
b.
Insufficient fluorine intake with tea
c.
Excessive fluorine levels in water
d.
Insufficient fluorine levels in soil and water
e.
Excessive fluorine levels in vegetable products
Коментар
Правильна відповідь: Excessive fluorine levels in water
Питання 27
Відповіді не було
Макс. оцінка до 1,00

Відмітити питання
Текст питання
A 36-year-old patient complains of suffocation attacks with predominantly problematic
exhalation. The attacks occur up to 2--3 times a day and can be relieved by inhalation of
β₂-adrenomimetics. This condition lasts for 10 years already. Objectively, the chest is
expanded, percussion detects a bandbox resonance over the lungs, auscultation detects
harsh respiration with prolonged exhalation. Laboratory analysis of sputum detects
numerous eosinophils, Charcot-Leyden crystals, and Curschmann spirals. What is the
provisional diagnosis in this case?
Питання 27Відповідь
a.
Chronic obstructive bronchitis
b.
Bronchial asthma
c.
Carcinoid syndrome
d.
Spontaneous pneumothorax
e.
Cardiac asthma
Коментар
Правильна відповідь: Bronchial asthma
Питання 28
Відповіді не було
Макс. оцінка до 1,00

Відмітити питання
Текст питання
On examination a man presents with the following signs: progressing weakness, loss of
working ability, rapid physical and mental fatigability, sensations of cold and hunger, and
weight loss. What type of alimentary disorder is it?
Питання 28Відповідь
a.
Protein-energy undernutrition
b.
Dietary fiber deficiency
c.
Polyunsaturated fatty acid deficiency
d.
Mineral deficiency
e.
Vitamin deficiency
Коментар
Правильна відповідь: Protein-energy undernutrition
Питання 29
Відповіді не було
Макс. оцінка до 1,00

Відмітити питання
Текст питання
A newborn has a round red formation in the suprapubic region. Examination shows that
urine is being discharged in pulses from the two orifices located in the lower part of this
formation. Name this developmental anomaly:
Питання 29Відповідь
a.
Urachal cyst
b.
Vesico-umbilical fistula
c.
Bladder exstrophy
d.
Bladder diverticulum
e.
Bladder agenesis
Коментар
Правильна відповідь: Bladder exstrophy
Питання 30
Відповіді не було
Макс. оцінка до 1,00

Відмітити питання
Текст питання
A 49-year-old man complains of angina pectoris attacks that occur when he walks up to
500 m. He has a many-year-long history of chronic bronchitis. Examination detects a
small number of dry diffuse crackles in the lungs, the respiratory rate is 18/min. The
borders of the heart are expanded to the left, the heart sounds are muffled, the heart
rate=PS=86/min., the blood pressure is 160/100 mm Hg. Complete blood count shows
the following: Hb --- 160 g/L, leukocytes --- $6.4 · 10^9$/L, ESR --- 7 mm/hour. ECG
shows hypertrophy of the left ventricle. What group of drugs is contraindicated in this
case, taking into account the concomitant pathology?
Питання 30Відповідь
a.
Antiplatelet drugs
b.
Beta blockers
c.
Calcium antagonists
d.
Angioprotectors
e.
Long-acting nitrates
Коментар
Правильна відповідь: Beta blockers
Питання 31
Відповіді не було
Макс. оцінка до 1,00

Відмітити питання
Текст питання
A 38-year-old patient complains of pain in the area of the metatarsophalangeal joints of
toes 1--2 on the right foot. Examination detects bluish-purple skin over the affected
joints that is hot to the touch. There are nodular formations covered with thin shiny skin
in the area of the auricles. What drug must be prescribed for this patient as a part of the
urate-lowering therapy?
Питання 31Відповідь
a.
Febuxostat
b.
Colchicine
c.
Nimesulide
d.
Methotrexate
e.
Allopurinol
Коментар
Правильна відповідь: Allopurinol
Питання 32
Відповіді не було
Макс. оцінка до 1,00

Відмітити питання
Текст питання
A 32-year-old patient suffers from epilepsy. Suddenly, without any cause, the patient
became excited. Objectively, the patient is disoriented in the personal identity and
environment, has visual and auditory hallucinations of threatening content and
delusional ideas of reference and persecution. There are expressions of fear, anger, and
rage on the patient's face. The patient's behavior is aggressive, accompanied by
destructive actions. What psychopathological syndrome is it?
Питання 32Відповідь
a.
Delirium syndrome
b.
Hebephrenic syndrome
c.
Twilight state
d.
Manic syndrome
e.
Paranoid syndrome
Коментар
Правильна відповідь: Twilight state
Питання 33
Відповіді не було
Макс. оцінка до 1,00

Відмітити питання
Текст питання
A 55-year-old woman complains of deformed finger joints and pain in them during
movements. Objectively, there are small bony formations on the lateral surfaces of the
distal interphalangeal joints. The formations are slightly painful to palpation. What are
they called?
Питання 33Відповідь
a.
Heberden's nodes
b.
Erythema nodosum
c.
Tophi
d.
Rheumatoid nodules
e.
Bouchard's nodes
Коментар
Правильна відповідь: Heberden's nodes
Питання 34
Відповіді не було
Макс. оцінка до 1,00

Відмітити питання
Текст питання
In the computer lab of the research sector at a polytechnic institute, the workstations of
laboratory technicians are located close in front of the screens. Throughout their whole
working day, the technicians are exposed to the electromagnetic waves of ultra high
frequencies. Exposure to intense electromagnetic waves of such frequency is especially
dangerous for the:
Питання 34Відповідь
a.
Hearing analyzer
b.
Pain sensitivity
c.
Sensory sensitivity
d.
Tactile sensitivity
e.
Visual analyzer
Коментар
Правильна відповідь: Visual analyzer
Питання 35
Відповіді не було
Макс. оцінка до 1,00

Відмітити питання
Текст питання
A food poisoning occurred in a canteen, affecting 15 people. The clinical presentation of
the poisoning indicates its staphylococcal etiology. What biological materials, obtained
from the patients, must be sent for laboratory testing to confirm food poisoning?
Питання 35Відповідь
a.
Vomitus
b.
Urine for clinical analysis
c.
Blood for hemoculture
d.
Saliva
e.
Blood for clinical analysis
Коментар
Правильна відповідь: Vomitus
Питання 36
Відповіді не було
Макс. оцінка до 1,00

Відмітити питання
Текст питання
A 57-year-old man complains of cough with profuse mucopurulent sputum (up to 150
mL per 24 hours). Objectively, he has drumstick fingers with watch-glass nails. Percussion
produces a shortened sound over the lungs. Auscultation detects moderate and large
bubbling crackles. Complete blood count shows leukocytosis and a left shift in the
leukogram. Chest X-ray shows intensified pulmonary pattern and ring-like shadows.
Bronchography detects multiple cylindrical thickenings of the bronchi with clear margins.
Make the provisional diagnosis:
Питання 36Відповідь
a.
Chronic pleural empyema
b.
Pulmonary gangrene
c.
Pulmonary echinococcosis
d.
Pulmonary cyst
e.
Bronchiectasis
Коментар
Правильна відповідь: Bronchiectasis
Питання 37
Відповіді не було
Макс. оцінка до 1,00
Відмітити питання
Текст питання
A four-month-old child suddenly became ill. The following symptoms has appeared: an
increase of the body temperature to $38,5^oC$, one-time vomiting, lethargy. In10 hours,
on the buttocks and lower limbs appeared a rash in the form of petechie, spots and
pustules. Some hemorrhagic elements are with necrosis in the center. What disease is
most likely present in this child?
Питання 37Відповідь
a.
Scarlet fever
b.
Meningococcemia
c.
Rubella
d.
Flu
e.
Hemorrhagic vasculitis
Коментар
Правильна відповідь: Meningococcemia
Питання 38
Відповіді не було
Макс. оцінка до 1,00

Відмітити питання
Текст питання
Examination of a victim of a traffic accident detects cyanosis and problems with
breathing. The patient is in a severe condition, the right half of the chest lags behind in
the act of breathing, the intercostal spaces are widened on the right, a bandbox
resonance is observed during percussion, breathing cannot be heard during auscultation.
What is the most likely diagnosis in this case?
Питання 38Відповідь
a.
Valvular pneumothorax
b.
Total hemothorax on the right
c.
Acute purulent pleurisy
d.
Pneumoperitoneum
e.
Open pneumothorax
Коментар
Правильна відповідь: Valvular pneumothorax
Питання 39
Відповіді не було
Макс. оцінка до 1,00

Відмітити питання
Текст питання
A 27-year-old woman, gravida 1, para 1, was hospitalized into the maternity ward. She
had a 3-year-long history of primary infertility. Contractions started 9 hours ago, occur
every 4--5 minutes, and last 20--25 seconds. The waters broke 2.5 hours ago. The fetal
heartbeat is 136/min. The small segment of the fetal head lies in the plane of the inlet
into the lesser pelvis. The cervix is smoothed out, its opening is 4 cm. The amniotic sac is
absent. What complication occurred during the childbirth?
Питання 39Відповідь
a.
Secondary weakness of labor activity
b.
Primary weakness of labor activity
c.
Pathological preliminary period
d.
Normal labor activity
e.
Discoordinated labor activity
Коментар
Правильна відповідь: Primary weakness of labor activity
Питання 40
Відповіді не було
Макс. оцінка до 1,00

Відмітити питання
Текст питання
A 51-year-old man after an overexposure to cold has developed an acute pain in his
lower abdomen and a burning pain that occurs at the end of urination. Urination occurs
up to 15 times per 24 hours. The urine is turbid and contains blood. Clinical urinalysis
shows leukocytes in the whole vision field and isolated erythrocytes. What provisional
diagnosis can be made?
Питання 40Відповідь
a.
Acute pyelonephritis
b.
Acute glomerulonephritis
c.
Acute cystitis
d.
Urolithiasis
e.
Acute urethritis
Коментар
Правильна відповідь: Acute cystitis
Питання 41
Відповіді не було
Макс. оцінка до 1,00

Відмітити питання
Текст питання
A 55-year-old woman came to a doctor with complaints of thyroid enlargement that
could be observed throughouth the last 2 years and a discomfort during swallowing.
Objectively, she has signs of hypothyroidism. The thyroid gland on palpation is dense,
non-fused with the surrounding tissues and mobile during swallowing. The regional
lymph nodes are not enlarged. Laboratory testing detects antithyroid antibodies in the
blood serum. What is the most likely diagnosis in this case?
Питання 41Відповідь
a.
Acute thyroiditis
b.
Hashimoto's thyroiditis
c.
Endemic goiter
d.
Thyroid cancer
e.
Midline cervical cyst
Коментар
Правильна відповідь: Hashimoto's thyroiditis
Питання 42
Відповіді не було
Макс. оцінка до 1,00

Відмітити питання
Текст питання
A 30-year-old woman came to a doctor with complaints of menstruations that have been
missing for two years after her second childbirth.The childbirth was complicated by a
massive bleeding. After giving birth, the woman started noticing hair loss and weight
loss. Objectively, the woman's body type is asthenic, her external genitalia are
hypoplastic, the cervix is cylindrical, the body of the uterus is small and painless, the
uterine appendages cannot be detected. What is the most likely diagnosis in this case?
Питання 42Відповідь
a.
Uterine pregnancy
b.
Pituitary tumor (Cushing disease)
c.
Primary amenorrhea
d.
Pituitary amenorrhea (Sheehan syndrome)
e.
Stein-Leventhal syndrome
Коментар
Правильна відповідь: Pituitary amenorrhea (Sheehan syndrome)
Питання 43
Відповіді не було
Макс. оцінка до 1,00

Відмітити питання
Текст питання
A 74-year-old woman came to a doctor complaining of a pain in her right inguinal
region. The signs appeared suddenly, approximately 2 hours ago. The woman notes that
she already had these signs 3 weeks ago, but back then they disappeared on their own
after she lay down. Objectively, palpation detects below the Poupart's ligament a sharply
painful, dense, and tense formation 3.5 cm in diameter. The Dejerine sign (aggravation
on coughing) is negative. What is the most likely diagnosis in this case?
Питання 43Відповідь
a.
Acquired incarcerated inguinal hernia
b.
Acquired strangulated inguinal hernia
c.
Acquired strangulated femoral hernia
d.
Acquired incarcerated femoral hernia
e.
Inguinal lymphadenitis
Коментар
Правильна відповідь: Acquired strangulated femoral hernia
Питання 44
Відповіді не було
Макс. оцінка до 1,00

Відмітити питання
Текст питання
A 42-year-old man complains of weakness, palpitations, nosebleeds, and skin
hemorrhages. His condition progressively deteriorates throughout the last month.
Objectively, his condition is severe, he has petechial and spotted hemorrhages on the
skin of his limbs and torso, lymph nodes and spleen are not palpable, the pulse is
116/min., the liver is +2 cm. Complete blood count reveals pancytopenia. What disease
can be primarily suspected in this case?
Питання 44Відповідь
a.
Acute agranulocytosis
b.
Hypoplastic anemia
c.
Werlhof disease
d.
Hemorrhagic vasculitis
e.
Acute leukemia
Коментар
Правильна відповідь: Hypoplastic anemia
Питання 45
Відповіді не було
Макс. оцінка до 1,00

Відмітити питання
Текст питання
Several chemical substances enter the human body from atmospheric air. What is the
type of joint action, where the overall effect on the human body is more than the sum of
the individual effects of each separate substance included in the combination?
Питання 45Відповідь
a.
Isolated action
b.
Antagonism
c.
Complex action
d.
Combined action
e.
Potentiation
Коментар
Правильна відповідь: Potentiation
Питання 46
Відповіді не було
Макс. оцінка до 1,00

Відмітити питання
Текст питання
A man was repairing a fence and injured his right elbow with a wire. Three days later he
developed edema, hyperemia, reduced range of movements in the joint, intense pain,
and local and general increase of temperature. Objectively, the right elbow is enlarged,
the skin over it is edematous and hyperemic; regional lymph nodes are enlarged,
thickened, and painful. Palpation of the right elbow detects sharp pain and fluctuation.
Make the provisional diagnosis.
Питання 46Відповідь
a.
Hidradenitis
b.
Erysipelas
c.
Abscess
d.
Bursitis
e.
Lymphadenitis
Коментар
Правильна відповідь: Bursitis
Питання 47
Відповіді не було
Макс. оцінка до 1,00

Відмітити питання
Текст питання
One week ago, a 68-year-old woman suddenly developed pain in the left half of her
chest and shortness of breath. Objectively, the following is observed: cyanosis, distended
neck veins, pulse --- 100/min., blood pressure --- 110/70 mm Hg, respiratory rate ---
28/min., liver +4 cm, the left lower leg is edematous and sharply painful to palpation.
Auscultation detects a dull lung sound on the left, below the scapula; sonorous moist
fine vesicular crackles are present; the borders of the heart are expanded on the right;
the II heart sound is accentuated over the pulmonary artery. ECG shows deep S waves in
leads I and aVL, as well as deep Q waves in leads III and aVF, and a negative T wave in
leads III and aVF. What is the most likely diagnosis in this case?
Питання 47Відповідь
a.
Pericarditis
b.
Pleurisy
c.
Croupous pneumonia
d.
Thromboembolism of pulmonary artery branches
e.
Myocardial infarction
Коментар
Правильна відповідь: Thromboembolism of pulmonary artery branches
Питання 48
Відповіді не було
Макс. оцінка до 1,00

Відмітити питання
Текст питання
An 18-year-old young man during hospitalization complains of general weakness, body
temperature of 37.5°C, loss of appetite, nausea, heaviness in the right subcostal region,
and discolored stool and urine. The disease onset was 5 days ago. Objectively, his skin
and sclerae are slightly icteric. The liver protrudes by 3 cm and is tender to palpation.
Patient's urine is dark brown, while his stool is light-colored. He usually drinks water from
a tap. What is the most likely diagnosis in this case?
Питання 48Відповідь
a.
Leptospirosis
b.
Malaria
c.
Viral hepatitis A
d.
Viral hepatitis B
e.
Typhoid fever
Коментар
Правильна відповідь: Viral hepatitis A
Питання 49
Відповіді не було
Макс. оцінка до 1,00

Відмітити питання
Текст питання
A 23-year-old woman with type 1 diabetes mellitus during the 2nd week of community-
acquired pneumonia developed nausea and vomiting. In the evening she has lost her
consciousness and was hospitalized. Objectively, the patient's skin is pale and dry. Her
respiration is loud, the tongue is dry, with brown deposit. Her heart rate is 129/min.,
blood pressure is 85/50 mm Hg. Palpation of the patient's abdomen provokes no
response. The liver is +3 cm. Acetone test is markedly positive, blood glucose is 26
mmol/L. Make the provisional diagnosis:
Питання 49Відповідь
a.
Hyperosmolar coma
b.
Hepatic coma
c.
Lactacidemic coma
d.
Ketoacidotic coma
e.
Infectious toxic shock
Коментар
Правильна відповідь: Ketoacidotic coma
Питання 50
Відповіді не було
Макс. оцінка до 1,00

Відмітити питання
Текст питання
For the last 2 years, a 32-year-old woman has been observing periodical pain attacks in
her right subcostal area that could be removed with no-spa (drotaverine). The pain is not
always associated with meals, sometimes it is caused by anxiety and accompanied by
cardiac pain and palpitations. Objectively, the woman is emotionally labile. Abdominal
palpation detects a slight pain in the area of the gallbladder. What pathology is the most
likely to cause such clinical presentation?
Питання 50Відповідь
a.
Chronic cholangitis
b.
Chronic pancreatitis
c.
Duodenitis
d.
Chronic cholecystitis
e.
Biliary dyskinesia
Коментар
Правильна відповідь: Biliary dyskinesia
Питання 51
Відповіді не було
Макс. оцінка до 1,00

Відмітити питання
Текст питання
A 20-year-old woman complains of headaches, vertigo, tearfulness, vomiting, pain in the
area of the heart, and tachycardia. The signs appear 6-7 days before menstruation and
disappear in the first days of menstruation. Make the diagnosis:
Питання 51Відповідь
a.
Diencephalic syndrome
b.
Stein-Leventhal syndrome
c.
Metabolic craniopathy
d.
Algomenorrhea
e.
Premenstrual syndrome
Коментар
Правильна відповідь: Premenstrual syndrome
Питання 52
Відповіді не було
Макс. оцінка до 1,00

Відмітити питання
Текст питання
A 58-year-old man had a moderate ischemic stroke in the right hemisphere of the brain
10 days ago. At the time of hospitalization, the deficit score on the NIHSS scale was 9
points. He suffers from moderate arterial hypertension, atrial fibrillation, urolithiasis, and
gout (remission). What must be prescribed to the patient for secondary prevention of
stroke?
Питання 52Відповідь
a.
High doses of lipid-lowering agents (e.g., atorvastatin --- 80 mg per day)
b.
Aspirin, 100 mg per day
c.
Oral anticoagulants (warfarin or NOACs --- dabigatran, rivaroxaban, apixaban)
d.
Drugs that improve cerebral blood flow
e.
Clopidogrel, 75 mg per day
Коментар
Правильна відповідь: Oral anticoagulants (warfarin or NOACs --- dabigatran,
rivaroxaban, apixaban)
Питання 53
Відповіді не було
Макс. оцінка до 1,00

Відмітити питання
Текст питання
A 30-year-old man, a cable presser, complains of inertness, memory problems, and pain
in his limbs. Objectively, he presents with skin pallor, anemia, reticulocytosis, basophilic
stippling of erythrocytes, and high levels of porphyrin in urine. This man has the signs of
the following disease:
Питання 53Відповідь
a.
Asbestosis
b.
Berylliosis
c.
Siderosis
d.
Mercurialism
e.
Saturnism
Коментар
Правильна відповідь: Saturnism
Питання 54
Відповіді не було
Макс. оцінка до 1,00

Відмітити питання
Текст питання
A 7-year-old boy after playing with a cat suddenly developed problems with breathing.
Objectively, the boy is pale, frightened, and sits, leaning onto his arms. His body
temperature is 36.6°C, heart rate -- 120/min., respiration rate -- 42/min., speaks in
syllables. The exhale is long and accompanied by wheezing. Percussion detects a
bandbox resonance over the lungs. An inhalation of ventolin (salbutamol) solution was
started via a nebulizer. What can help assess the child's need for oxygen therapy in this
case?
Питання 54Відповідь
a.
Chest X-ray
b.
Assessment of accessory respiratory \\ muscles involvement
c.
Spirometry
d.
Pulse oximetry
e.
Pneumotachometry
Коментар
Правильна відповідь: Pulse oximetry
Питання 55
Відповіді не було
Макс. оцінка до 1,00

Відмітити питання
Текст питання
A 6-year-old child developed a fever of 37.5°C, stuffed nose with a small amount of
mucous discharge, and a scratchy sensation in the throat. On the fourth day after the
onset of the disease, the temperature normalized but complaints of pain in the legs
appeared. The child started limping and dragging the left leg. Muscle tone and reflexes
are decreased, while sensitivity is retained. What disease can be suspected in this case?
Питання 55Відповідь
a.
Polyradiculoneuritis
b.
Botulism
c.
Diphtheritic polyneuritis
d.
Paralytic poliomyelitis
e.
Influenza with associated encephalopathy
Коментар
Правильна відповідь: Paralytic poliomyelitis
Питання 56
Відповіді не було
Макс. оцінка до 1,00

Відмітити питання
Текст питання
A man complains of a heaviness behind his sternum, periodical sensation of food
retention, and dysphagia. During X-ray the barium contrast reveals a single pouch-like
protrusion in the right anterior wall of the esophagus. The protrusion has clear margins
and a clearly defined neck. What is the most likely diagnosis in this case?
Питання 56Відповідь
a.
Hiatal hernia
b.
Esophageal carcinoma
c.
Esophageal diverticulum
d.
Varicose veins of the esophagus
e.
Esophageal polyp
Коментар
Правильна відповідь: Esophageal diverticulum
Питання 57
Відповіді не було
Макс. оцінка до 1,00

Відмітити питання
Текст питання
A 42-year-old man, a miner, was freed after 12 hours spent under the rubble. Objectively,
his both lower legs and feet are pale; there is no pulsation on the peripheral vessels, no
sensitivity, and no passive movements in the joints. He was hospitalized with tourniquets
applied to both limbs. What urgent first aid must be provided in this case to prevent
myoglobinuria and acute kidney failure?
Питання 57Відповідь
a.
Remove the tourniquets, provide detoxification therapy
b.
Do not remove the tourniquets, provide detoxification therapy
c.
Do not remove the tourniquets, amputate the limbs above the tourniquets
d.
Remove the tourniquets, provide hyperbaric oxygenation
e.
Do not remove the tourniquets, provide hyperbaric oxygenation
Коментар
Правильна відповідь: Do not remove the tourniquets, amputate the limbs above the
tourniquets
Питання 58
Відповіді не було
Макс. оцінка до 1,00

Відмітити питання
Текст питання
A 37-year-old patient was repeatedly treated in a psychiatric hospital. The current
hospitalization was due to behavioral disorders in the form of reticence, refusal to eat,
periodical chaotic excitement with stereotyped movements. During the examination, the
patient is sluggish, remains in bed, and no contact with him can be made. He does not
answer any questions, his position is monotonous, he is hypomimic. The "proboscis"
sign, the waxy flexibility of the muscles, and the "psychological pillow" sign are observed.
This condition lasts for a week already. What is the provisional diagnosis in this case?
Питання 58Відповідь
a.
Simple-type schizophrenia
b.
Reactive stupor
c.
Schizophrenia with paranoia
d.
Hebephrenic schizophrenia
e.
Catatonic schizophrenia
Коментар
Правильна відповідь: Catatonic schizophrenia
Питання 59
Відповіді не було
Макс. оцінка до 1,00

Відмітити питання
Текст питання
A woman complains of frequent and copious foamy stools without pathological
admixtures, cramp-like pain in her periumbilical region, rumbling in the stomach, and
fever. From her history, it is known that she was eating soft-boiled duck eggs. What is
the most likely causative agent of her condition?
Питання 59Відповідь
a.
Salmonella typhi
b.
Escherichia coli
c.
Salmonella enteritidis
d.
V. cholerae
e.
Shigella sonnei
Коментар
Правильна відповідь: Salmonella enteritidis
Питання 60
Відповіді не було
Макс. оцінка до 1,00

Відмітити питання
Текст питання
An 18-year-old young man came to a doctor with complaints of cramping abdominal
pain, frequent liquid stools with mucus and fresh blood. The symptoms have been
observed for the last four months, during which the patient lost 10 kg. Physical
examination shows soft abdomen, painful on the left along the large intestine; the
sigmoid colon is spastic. Complete blood count shows the following: erythrocytes ---
$3.2 · 10^{12$/L, hemoglobin --- 92 g/L, leukocytes --- $10.6 · 10^9$/L, ESR --- 34
mm/hour. Bacteriological culture of feces shows negative results. Colonoscopy revealed
inflammation of the large intestine with the loss of vascular pattern, as well as an area of
increased bleeding with clear boundaries. What treatment tactics should be chosen for
this patient?
Питання 60Відповідь
a.
Repeated collection of a stool sample for bacteriological testing
b.
Prescription of prednisolone and mesalamine (mesalazine)
c.
Prescription of metronidazole
d.
Transfusion of packed red blood cells
e.
Surgical treatment
Коментар
Правильна відповідь: Prescription of prednisolone and mesalamine (mesalazine)
Питання 61
Відповіді не було
Макс. оцінка до 1,00

Відмітити питання
Текст питання
For intensive infusion therapy, a patient with acute respiratory failure underwent
subclavian vein catheterization using the Seldinger technique. After administration of
600 mL of the infusion solution, the patient's condition sharply deteriorated, tachypnea
increased from 26/min. to 40/min., tidal volume decreased from 400 mL to 250 mL.
Auscultation detects sharply weakened respiration on the right. Percussion detects a dull
sound. What complication developed in this patient?
Питання 61Відповідь
a.
Hydrothorax
b.
Pulmonary embolism
c.
Acute heart failure
d.
Cerebral edema
e.
Pulmonary edema
Коментар
Правильна відповідь: Hydrothorax
Питання 62
Відповіді не було
Макс. оцінка до 1,00

Відмітити питання
Текст питання
Within several days, a person developed a reduction in the peripheral vision, resembling
a flap, on the outer side. The patient does not know the cause of this condition. The eye
is calm, the optical media are transparent. On the nasal side of eye, ophthalmoscopy
detects a gray "sail" with vessels that wavers during eye movements. The optic disc and
blood vessels are without changes. What is the most likely diagnosis in this case?
Питання 62Відповідь
a.
Hemianopsia with a neurological pathology
b.
Retinal detachment
c.
Vascular pathology of the retina
d.
Lens subluxation
e.
Initial signs of glaucoma
Коментар
Правильна відповідь: Retinal detachment
Питання 63
Відповіді не було
Макс. оцінка до 1,00

Відмітити питання
Текст питання
What is the type of joint action, where the overall effect on the human body is less than
the sum of the individual effects of each separate substance included in the
combination?
Питання 63Відповідь
a.
Potentiation
b.
Combined action
c.
Isolated action
d.
Antagonism
e.
Complex action
Коментар
Правильна відповідь: Antagonism
Питання 64
Відповіді не було
Макс. оцінка до 1,00

Відмітити питання
Текст питання
A man complains of red and sore skin on his right calf. Objectively, his body temperature
is 38.5°С; his inguinal lymph nodes on the right are enlarged and painful; the skin is red,
with clear boundaries between the redness and healthy skin; edema is observed; there
are vesicles filled with a dark liquid; palpation is painful. What is the provisional diagnosis
in this case?
Питання 64Відповідь
a.
Herpetic infection
b.
Cutaneous anthrax
c.
Varicella
d.
Erysipelas
e.
Phlegmon of the leg
Коментар
Правильна відповідь: Erysipelas
Питання 65
Відповіді не було
Макс. оцінка до 1,00

Відмітити питання
Текст питання
A 36-year-old woman complains of nausea, belching, liquid stool, and a pain in the
epigastrium after meals. For the last 2 years the disease has been slowly progressing.
Objectively, her skin is pale and dry, her tongue is coated, moist, and has imprints of the
teeth on its edges. Abdominal palpation detects a diffuse pain in the epigastrium. What
test will be the most informative in this case and should be conducted next?
Питання 65Відповідь
a.
Abdominal CT scan
b.
Fibrogastroscopy with biopsy of the \\ gastric mucosa
c.
Gastrointestinal X-ray
d.
Comprehensive complete blood count
e.
Fractional analysis of gastric secretion
Коментар
Правильна відповідь: Fibrogastroscopy with biopsy of the \\ gastric mucosa
Питання 66
Відповіді не було
Макс. оцінка до 1,00

Відмітити питання
Текст питання
A 54-year-old man was hospitalized with complaints of general weakness, fever of
38.6°С, shortness of breath during significant physical exertion, and frequent nosebleeds.
He works in production of plastics, where he comes into contact with aromatic
compounds. Objectively, his skin is pale and dry. Complete blood count shows the
following: erythrocytes --- $2.1 · 10^{12$/L, Hb --- 90 g/L, leukocytes --- $2.2 · 10^9$/L,
eosinophils --- 1 %, band neutrophils --- 1 %, segmented neutrophils --- 75 %,
lymphocytes --- 20 %, myelocytes --- 3 %, platelets --- $30 · 10^9$/L, ESR --- 32
mm/hours. What is the most likely diagnosis in this case?
Питання 66Відповідь
a.
Chronic nitrobenzene intoxication
b.
Chronic benzene intoxication
c.
Chronic aniline intoxication
d.
Chronic lead intoxication
e.
Chronic tetraethyllead intoxication
Коментар
Правильна відповідь: Chronic benzene intoxication
Питання 67
Відповіді не було
Макс. оцінка до 1,00

Відмітити питання
Текст питання
A 22-year-old young man came to a doctor with complaints of fever and cough with
yellow sputum. These symptoms have been observed for the last seven days. Physical
examination shows the following: body temperature --- 38.3°C, blood pressure ---
130/70 mm Hg, heart rate --- 79/min., respiratory rate --- 17/min., SpO₂ --- 95 %.
Auscultation detects moist crackles on the left. Survey X-ray of the chest reveals left
lower lobe consolidations. What drug should be prescribed to the patient in this case?
Питання 67Відповідь
a.
Levofloxacin
b.
Ciprofloxacin
c.
Azithromycin
d.
Gentamicin
e.
Trimethoprim/sulfamethoxazole
Коментар
Правильна відповідь: Azithromycin
Питання 68
Відповіді не було
Макс. оцінка до 1,00

Відмітити питання
Текст питання
A newborn boy was brought into a hospital on the second day after the onset of the
disease. His parents complain of his fussiness, regurgitation, fever of 38.5°C, red skin with
infiltration in the area of his lower back. Objectively, the child is in a severe condition,
inert, pale, suckles poorly. In the lumbar region, on the sacrum, and on the buttocks
there is an infiltration with hyperemic areas and a soft spot 8x7 cm in its center. The stool
is 10 times per 24 hours with green coloring and mucus. What is the most likely
diagnosis in this case?
Питання 68Відповідь
a.
Hemangioma
b.
Congenital soft-tissue tumor
c.
Phlegmon of the newborn
d.
Adiponecrosis
e.
Erysipelas
Коментар
Правильна відповідь: Phlegmon of the newborn
Питання 69
Відповіді не було
Макс. оцінка до 1,00

Відмітити питання
Текст питання
Screening detected phenylketonuria in a two-week-old baby. What treatment must be
prescribed for this child to prevent severe complications in the future?
Питання 69Відповідь
a.
Sunbathing
b.
Antibiotic therapy
c.
Vitamin therapy
d.
Hormone therapy
e.
Special diet
Коментар
Правильна відповідь: Special diet
Питання 70
Відповіді не було
Макс. оцінка до 1,00

Відмітити питання
Текст питання
A 22-year-old pregnant woman was hospitalized in a severe condition. Throughout the
past three days, she developed edemas, headache, nausea, and one episode of vomiting.
Objectively, her consciousness is clouded, her blood pressure is 160/130 mm Hg. She
presents with small fibrillar twitching of the facial muscles and problems with nasal
breathing. During transportation, the woman's arms started twitching, her body
stretched out, her spine curved, her jaws tightly clenched, and she stopped breathing.
Then she developed clonic seizures and marked cyanosis. After that, the seizures
stopped, a deep noisy inhale occurred, and blood-stained foam appeared on the
patient's lips. What is the most likely diagnosis in this case?
Питання 70Відповідь
a.
Hypertensive crisis
b.
Eclampsia
c.
Epilepsy
d.
Diabetic coma
e.
Chorea
Коментар
Правильна відповідь: Eclampsia
Питання 71
Відповіді не було
Макс. оцінка до 1,00

Відмітити питання
Текст питання
A 40-year-old man was brought into a hospital with a closed chest trauma. Examination
shows that the right side of his chest lags behind during breathing. Palpation detects a
sharp pain in the projection of ribs V, VI, and VII on the anterior axillary line and
subcutaneous emphysema on the right side of the torso. Percussion reveals tympanitis
over the right-sided segments of the chest. Auscultation detects no breathing on the
right and vesicular breathing on the left. What surgical procedure is necessary for this
patient?
Питання 71Відповідь
a.
Tight bandaging of the chest
b.
Drainage of the right pleural cavity
c.
Drainage of subcutaneous emphysema
d.
Splint stabilization of the rib fracture
e.
Immediate thoracotomy
Коментар
Правильна відповідь: Drainage of the right pleural cavity
Питання 72
Відповіді не було
Макс. оцінка до 1,00

Відмітити питання
Текст питання
A 45-year-old man came to a doctor complaining of a sore throat and fever that last for
ten days. The patient has a history of ulcerative colitis, is a smoker, and has been
smoking one pack of cigarettes per day for ten years. The patient's father died of cancer
of the large intestine at the age of 50. Physical examination shows the following: body
temperature --- 38.6°C, blood pressure --- 130/80 mm Hg, pulse --- 72/min., respiratory
rate --- 18/min. Examination of the oral cavity reveals erythematous pharynx and
exudate in the tonsillar crypts. Antigen test for group A streptococcus was positive. What
procedure that doctor can recommend for secondary prevention of cancer would be
optimal for this patient?
Питання 72Відповідь
a.
Fecal occult blood test
b.
PSA test and digital rectal examination
c.
Flexible sigmoidoscopy
d.
Colonoscopy
e.
Low-dose CT scan of the lungs
Коментар
Правильна відповідь: Colonoscopy
Питання 73
Відповіді не було
Макс. оцінка до 1,00

Відмітити питання
Текст питання
A 32-year-old man has a closed cerebrocranial trauma, a closed chest trauma, and a
closed right femoral fracture. His blood pressure was 100/60 mm Hg, pulse -- 124/min.,
respiration rate -- 28/min. Two hours after the skeletal traction was performed under
topical lidocaine anesthesia, the patient's condition suddenly became worse. His face
and neck became cyanotic, blood pressure -- 60/40 mm Hg, heart rate -- 160/min.,
respiration rate -- 44/min. What complication has likely developed in this case?
Питання 73Відповідь
a.
Acute posthemorrhagic anemia
b.
Pain shock
c.
Pulmonary embolism
d.
Fat pulmonary embolism
e.
Myocardial infarction, cardiogenic shock
Коментар
Правильна відповідь: Fat pulmonary embolism
Питання 74
Відповіді не було
Макс. оцінка до 1,00

Відмітити питання
Текст питання
A worker at a workshop that produces car batteries came to a doctor with complaints of
nausea, loss of appetite, sharp pain in the abdominal cavity, and constipations.
Examination reveals elevated blood pressure, bradycardia, an enlarged liver, pain in the
right subcostal region, a grayish-blue stripe on the gums, and gray skin. Complete blood
count indicates the presence of erythrocytes with basophilic stippling and reduced
hemoglobin levels. Aminolevulinic acid and coproporphyrin can be detected in the
patient's urine. What is the most likely provisional diagnosis in this case?
Питання 74Відповідь
a.
Mercury poisoning
b.
Lead poisoning
c.
Aluminum poisoning
d.
Cadmium poisoning
e.
Food poisoning
Коментар
Правильна відповідь: Lead poisoning
Питання 75
Відповіді не було
Макс. оцінка до 1,00

Відмітити питання
Текст питання
A 64-year-old man is undergoing outpatient treatment for ischemic heart disease,
diffuse cardiosclerosis, persistent atrial fibrillation, and functional class III heart failure.
The pharmacotherapy consists of cordarone (amiodarone) twice a day, torasemide every
other day, and trimetazidine twice a day. The family physician recommended taking
warfarin (3 mg per day) to prevent thrombus formation. What parameter should be
measured in this case for optimal control of the effectiveness and safety of the
anticoagulant therapy?
Питання 75Відповідь
a.
Erythrocyte sedimentation rate
b.
Lee-White clotting time
c.
Platelet count
d.
Fibrinogen levels
e.
International normalized ratio
Коментар
Правильна відповідь: International normalized ratio
Питання 76
Відповіді не було
Макс. оцінка до 1,00

Відмітити питання
Текст питання
A 63-year-old man came to a doctor with complaints of marked general weakness, poor
appetite, weight loss, joint pain, and heaviness in the right subcostal area. Complete
blood count shows the following: erythrocytes --- $3.4 · 10^{12$/L, Hb --- 102 g/L, color
index --- 0.9, platelets --- $640 · 10^9$ /L, leukocytes --- $138 · 10^9$/L, blasts ---1 %,
promyelocytes --- 2 %, myelocytes --- 13 %, juvenile --- 12 %, band neutrophils --- 16 %,
segmented neutrophils --- 31 %, basophils --- 3 %, eosinophils --- 8 %, lymphocytes ---
9 %, monocytes --- 5 %, ESR --- 30 mm/hour. What is the provisional diagnosis?
Питання 76Відповідь
a.
Chronic lymphocytic leukemia
b.
Erythremia (polycythemia vera)
c.
Chronic myeloid leukemia
d.
Acute leukemia
e.
Leukemoid reaction
Коментар
Правильна відповідь: Chronic myeloid leukemia
Питання 77
Відповіді не було
Макс. оцінка до 1,00

Відмітити питання
Текст питання
A pregnant woman at 38 weeks of pregnancy was hospitalized into the maternity
hospital with complaints of a small amount of bloody discharge from the genital tracts
and a sharp pain in her entire abdomen that started one hour ago. No labor activity can
be detected. Objectively, the patient is pale, her pulse is 100/min., her blood pressure is
100/60 mm Hg, no fetal heartbeat. Palpation detects tense and painful uterus. Make the
diagnosis.
Питання 77Відповідь
a.
Acute kidney failure
b.
Uterine rupture
c.
Placenta previa
d.
Intranatal fetal death
e.
Placental abruption
Коментар
Правильна відповідь: Placental abruption
Питання 78
Відповіді не було
Макс. оцінка до 1,00

Відмітити питання
Текст питання
In a 70-year-old woman, chest fluorography shows a shadow of a heterogeneous
structure over the left dome of the diaphragm. X-ray with contrast detects the
abdominal segment of the esophagus in the chest cavity. What is the most likely
diagnosis in this case?
Питання 78Відповідь
a.
Hiatal hernia
b.
Benign esophageal tumor
c.
Esophageal diverticulum
d.
Esophagitis
e.
Achalasia cardia
Коментар
Правильна відповідь: Hiatal hernia
Питання 79
Відповіді не було
Макс. оцінка до 1,00

Відмітити питання
Текст питання
A patient underwent suture plication of the perforated duodenal ulcer. On the 3rd day
after the operation he started producing a large amount of discharge from the
abdominal drain tube. The discharge contains bile and has high amylase levels. What
complication occurred in the patient?
Питання 79Відповідь
a.
Acute cholecystitis
b.
Acute postoperative pancreatitis
c.
Hemorrhage from the ulcer
d.
Early postoperative adhesive obstruction
e.
Suture incompetence of the ulcerative defect
Коментар
Правильна відповідь: Suture incompetence of the ulcerative defect
Питання 80
Відповіді не було
Макс. оцінка до 1,00

Відмітити питання
Текст питання
A 20-year-old woman has been developing rhinitis and conjunctivitis in August-
September for the last 8 years. The last year during this period, attacks of bronchial
asthma started occurring as well. Skin testing detects hypersensitivity to Ambrosia
pollen. The antibodies that cause the exacerbation of this condition belong to the
following class of immunoglobulins:
Питання 80Відповідь
a.
Immunoglobulin A
b.
Immunoglobulin M
c.
Immunoglobulin E
d.
Immunoglobulin G
e.
Immunoglobulin D
Коментар
Правильна відповідь: Immunoglobulin E
Питання 81
Відповіді не було
Макс. оцінка до 1,00

Відмітити питання
Текст питання
A 7-year-old boy after a fall from a height presents with rapid and shallow breathing and
cyanotic face. The right half of his chest takes no part in the act of breathing. Percussion
detects tympanitis in this region, while auscultation detects no breathing there. What
pathology is the most likely cause of this clinical presentation? What instrumental
examination would be the most informative in this case?
Питання 81Відповідь
a.
Right-sided tension pneumothorax. Survey X-ray of the chest
b.
Tension cyst of the right lung. Tracheobronchoscopy
c.
Mediastinitis. Survey X-ray of the chest
d.
Right-sided hemothorax. Survey X-ray of the chest
e.
Airway foreign body. Diagnostic and therapeutic bronchoscopy
Коментар
Правильна відповідь: Right-sided tension pneumothorax. Survey X-ray of the chest
Питання 82
Відповіді не було
Макс. оцінка до 1,00

Відмітити питання
Текст питання
A 34-year-old woman came to a doctor with complaints of muscle weakness, thirst,
increased urination at night, paresthesias, and seizure attacks. Objectively, her general
condition is satisfactory, her face and legs are doughy, her pulse is 80/min., her blood
pressure is 200/110 mm Hg, the II heart sound is accentuated over the aorta. Complete
blood count shows K⁺ levels of 3.1 mmol/L and Na⁺ levels of 165 mmol/L. ECG shows
inversion of T waves and depression of S--T segments. Ultrasound detects hyperplasia of
the right adrenal gland. What is the provisional diagnosis in this case?
Питання 82Відповідь
a.
Primary hyperaldosteronism
b.
Glucosteroma
c.
Hypoparathyroidism
d.
Pheochromocytoma
e.
Essential hypertension
Коментар
Правильна відповідь: Primary hyperaldosteronism
Питання 83
Відповіді не було
Макс. оцінка до 1,00

Відмітити питання
Текст питання
A 25-year-old woman complains of pain in her right iliac region that lasts for 10--12 days
already and a menstruation delay of 7--8 weeks. Palpation detects pain in the right iliac
region. Gynecological examination detects pain in the right vaginal fornix, but no
enlargement of the uterus or ovaries. A right-sided ectopic pregnancy is suspected. What
examination method would be optimal in this case?
Питання 83Відповідь
a.
Ultrasound
b.
Thermography
c.
Pneumogynecography
d.
Pelvic X-ray
e.
Metrosalpingography
Коментар
Правильна відповідь: Ultrasound
Питання 84
Відповіді не було
Макс. оцінка до 1,00

Відмітити питання
Текст питання
A 64-year-old man came to a doctor with complaints of edemas of the legs, lower back,
and anterior abdominal wall. He has a history of chronic bronchitis with bronchiectasis.
Physical examination shows the following: body temperature --- 37.2°C, blood pressure -
-- 110/75 mm Hg, pulse --- 82/min., respiratory rate --- 19/min. General urinalysis results
are as follows: specific gravity --- 1025, protein --- 9.9 g/L, leukocytes --- 2--3 in the
vision field, erythrocytes --- 1--2 in the vision field, no casts. Daily proteinuria is 11.4 g
per 24 hours. Blood biochemistry test shows the following: total protein --- 52 g/L,
albumins --- 30 g/L, cholesterol --- 9.6 mmol/L. What type of kidney disorder is most
likely in this patient?
Питання 84Відповідь
a.
Pyelonephritis, urinary syndrome
b.
Glomerulonephritis, nephritic syndrome
c.
Diabetic nephropathy, nephritic syndrome
d.
Tubulointerstitial nephritis, nephrotic syndrome
e.
Amyloidosis, nephrotic syndrome
Коментар
Правильна відповідь: Amyloidosis, nephrotic syndrome
Питання 85
Відповіді не було
Макс. оцінка до 1,00
Відмітити питання
Текст питання
A 72-year-old patient came to a doctor with complaints of hand tremors, stiffness,
slowness of walking and speech, and difficulties in performing routine household chores.
Physical examination detects general bradykinesia, hypomimia, emprosthotonus,
"shuffling" gait, hand tremors at rest, D>S, plastic type of increased muscle tone, the
"cogwheel rigidity" phenomenon, and marked postural instability. What medicines must
be a primary part of the treatment regimen for this patient?
Питання 85Відповідь
a.
Medicines that improve microcirculation
b.
Anticholinesterase drugs
c.
Antidepressants
d.
Anticholinergic agents
e.
Medicines containing levodopa
Коментар
Правильна відповідь: Medicines containing levodopa
Питання 86
Відповіді не було
Макс. оцінка до 1,00

Відмітити питання
Текст питання
A pregnant woman suffers from essential hypertension of the first degree. At 35 weeks
of gestation, she developed edemas of the legs and anterior abdominal wall. Her 24-
hour urine protein is 5 g/L, the blood pressure increased to 170/120 mm Hg. She
developed a headache and her vision became worse. Four hours of intensive treatment
had no effect. What tactics would be necessary in this case?
Питання 86Відповідь
a.
Labor induction
b.
Immediate delivery by caesarean section
c.
Conservative management of the delivery
d.
Continuation of the intensive therapy
e.
Preparation of the cervix for preterm birth
Коментар
Правильна відповідь: Immediate delivery by caesarean section
Питання 87
Відповіді не було
Макс. оцінка до 1,00

Відмітити питання
Текст питання
A 3-year-old boy has been hospitalized with severe edemas. Objectively, the boy is pale,
his blood pressure is 90/60 mm Hg, he has oliguria. General urinalysis shows the
following: color --- yellow, specific gravity --- 1020, protein levels --- 3.5 g/L,
erythrocytes --- leached, 4--5 in the vision field, leukocytes --- 2--3 in the vision field,
24-hour proteinuria --- 6.6 g. The cholesterol levels are 9.8 mmol/L. What next step in
the patient's management would be optimal?
Питання 87Відповідь
a.
Monitoring throughout the next week
b.
Kidney biopsy
c.
Zimnitsky test
d.
Nechiporenko urine test
e.
Computed tomography without contrast
Коментар
Правильна відповідь: Kidney biopsy
Питання 88
Відповіді не було
Макс. оцінка до 1,00

Відмітити питання
Текст питання
A 32-year-old man has been hospitalized with fever episodes that are accompanied by
excessive sweating and occur every 48 hours. Twelve years ago he was in military service
in Tajikistan, where he had a case of malaria. Objectively, his sclerae are subicteric, his
skin is pale, the liver is dense and enlarged by 2 cm, the spleen is enlarged by 5 cm.
What test is necessary to clarify and confirm the diagnosis in this case?
Питання 88Відповідь
a.
Microscopy of the thick drop and blood smear
b.
Biochemical blood test
c.
Serum activity of ALT and AST enzymes
d.
Serological blood test
e.
Serum bilirubin levels
Коментар
Правильна відповідь: Microscopy of the thick drop and blood smear
Питання 89
Відповіді не було
Макс. оцінка до 1,00

Відмітити питання
Текст питання
A pediatrician needs to analyze the infant mortality rates. What must be taken as the unit
of observation for this purpose?
Питання 89Відповідь
a.
A case of death of a child after 28 days of life
b.
A case of the death of a child during childbirth
c.
A case of death of a child within the first 7 days of life
d.
A case of death of a child under one year of age
e.
A case of death of a child within the first month of life
Коментар
Правильна відповідь: A case of death of a child under one year of age
Питання 90
Відповіді не було
Макс. оцінка до 1,00

Відмітити питання
Текст питання
A 62-year-old man complains of constant pain in the epigastrium and weight loss of 12
kg. Physical and instrumental examinations (fibrogastroscopy with biopsy, abdominal
ultrasound, and chest X-ray) detected cancer of the body of the stomach without signs
of distant metastasis. Histology reveals moderately differentiated adenocarcinoma. What
scope of surgical intervention is advised in this case?
Питання 90Відповідь
a.
Ivor Lewis procedure
b.
Proximal subtotal gastric resection
c.
Distal subtotal gastric resection
d.
Gastroenteroanastomosis (gastroenteric bypass)
e.
Gastrectomy
Коментар
Правильна відповідь: Gastrectomy
Питання 91
Відповіді не було
Макс. оцінка до 1,00

Відмітити питання
Текст питання
On the fourth day of life, a healthy newborn child developed melena and started
vomiting blood. Coagulogram reveals prolonged prothrombin time, decreased
prothrombin index, and deficiency of plasma factors II, VII, IX, and X. What is the most
likely disease in this case?
Питання 91Відповідь
a.
Disseminated intravascular coagulation syndrome
b.
Hemorrhagic disease of the newborn
c.
Neonatal sepsis
d.
Hemophilia A
e.
Hemolytic disease of the newborn
Коментар
Правильна відповідь: Hemorrhagic disease of the newborn
Питання 92
Відповіді не було
Макс. оцінка до 1,00

Відмітити питання
Текст питання
A 34-year-old man fell ill 3 days ago after an overexposure to cold. He complains of a
fever of $39.2^o$С, marked general weakness, sweating, and cough. The cough was
initially dry, but within the last 24 hours a small amount of "rusty" sputum was produced.
Objective examination detects herpes on the lips. Percussion reveals a dull sound in the
lower pulmonary lobes. Auscultation detects bronchial breathing and tachycardia. No
changes were detected in the organs of the abdominal cavity. What is the most likely
diagnosis in this case?
Питання 92Відповідь
a.
Exudative pleurisy
b.
Community-acquired focal pneumonia
c.
Croupous pneumonia
d.
Lung abscess
e.
Nosocomial pneumonia
Коментар
Правильна відповідь: Croupous pneumonia
Питання 93
Відповіді не було
Макс. оцінка до 1,00

Відмітити питання
Текст питання
A 64-year-old patient with a tumor of the sigmoid colon and chronic thrombophlebitis
of the deep veins of the right leg is scheduled for a surgery. What is the optimal
medicine for prevention of deep vein thrombosis in this patient?
Питання 93Відповідь
a.
Rheopolyglucin (Dextran)
b.
Phenylinum (Phenindione)
c.
Low-molecular-weight heparin
d.
Regular heparin
e.
Acetylsalicylic acid
Коментар
Правильна відповідь: Low-molecular-weight heparin
Питання 94
Відповіді не було
Макс. оцінка до 1,00

Відмітити питання
Текст питання
A 40-year-old woman after an unsuccessful treatment by a neurologist was referred to a
gynecologist. She complains of swollen breasts, depression intermittent with agression,
weakness, tearfulness, numb arms, and meteorism that appear 2--3 days before a
menstruation and disappear after the menstruation is over. She considers herself ill for
the last 2 years. Gynecological examination detects no pathologic changes in her
genitals. She was diagnosed with premenstrual syndrome. What clinical form of
premenstrual syndrome is the most likely in this case?
Питання 94Відповідь
a.
Mixed
b.
Cephalgic
c.
Neuropsychic
d.
Crisis
e.
Edematous
Коментар
Правильна відповідь: Neuropsychic
Питання 95
Відповіді не було
Макс. оцінка до 1,00

Відмітити питання
Текст питання
A 38-year-old woman complains of a tensive pain in her lower abdomen and the small of
her back that is observed within the last month and intensifies on the day before
menstruation. Premenstrual dark bloody discharge was observed. She has a history of
four medical abortions and one birth. Ultrasound shows isolated foci of increased
echogenicity in the myometrium, increased anteroposterior size of the uterus, and round
hypoechogenic inclusions 2 mm in diameter. What is the most likely diagnosis in this
case?
Питання 95Відповідь
a.
Ovarian endometriosis
b.
Hormone-producing ovarian tumor
c.
Retrocervical endometriosis
d.
Adenomyosis
e.
Chorionepithelioma
Коментар
Правильна відповідь: Adenomyosis
Питання 96
Відповіді не було
Макс. оцінка до 1,00

Відмітити питання
Текст питання
A 25-year-old woman was hospitalized into the gynecological department with
complaints of pain in her lower abdomen and high temperature of 39.7°C) Objectively,
her blood pressure is 120/80 mm Hg, pulse -- 108/min., of satisfactory strength and
volume. The abdomen is moderately distended and sharply painful in its lower segments.
The Bloomberg's sign is positive in the hypogastrium. During vaginal examination, the
uterus and its appendages cannot be palpated because of anterior abdominal wall
rigidity. The posterior vaginal fornix is overhanging and sharply painful. What is the most
likely daignosis in this case?
Питання 96Відповідь
a.
Ovarian apoplexy
b.
Acute endometritis
c.
Acute adnexitis
d.
Pelvioperitonitis
e.
Ectopic pregnancy
Коментар
Правильна відповідь: Pelvioperitonitis
Питання 97
Відповіді не було
Макс. оцінка до 1,00

Відмітити питання
Текст питання
It was found that for every 100 births, women with risk factors had 30 preterm births,
while women without risk factors had 5 preterm births. What statistical method of data
processing would be optimal for a doctor to use to estimate the validity of differences
between the groups that are being compared?
Питання 97Відповідь
a.
Correlation analysis
b.
Calculation of relative values
c.
Calculation of the Student's criterion
d.
Standardization method
e.
Calculation of average values
Коментар
Правильна відповідь: Calculation of the Student's criterion
Питання 98
Відповіді не було
Макс. оцінка до 1,00

Відмітити питання
Текст питання
A full-term girl with chronic fetoplacental insufficiency was born from the second
pregnancy that risked termination at 25--27 weeks. During the childbirth, the umbilical
cord was revealed to be wrapped once around the neck of the baby, the extraction of
the shoulders was difficult. After the birth, the Apgar scale was used to evaluate the
baby's condition at the first minute of life: no breathing, the heart rate of 50/min., total
cyanosis, atony, areflexia. What would be the Apgar score in this case?
Питання 98Відповідь
a.
0 points
b.
4 points
c.
1 point
d.
2 points
e.
3 points
Коментар
Правильна відповідь: 1 point
Питання 99
Відповіді не було
Макс. оцінка до 1,00

Відмітити питання
Текст питання
Examination detected vesicles with seropurulent content on the neck, back of the head,
and buttocks of an infant on the 4th day of life. The patient's condition is satisfactory, the
child is active, all newborn reflexes can be fully induced, the umbilical cord is at the stage
of mummification, the umbilical area is without any peculiarities. What disease can be
suspected?
Питання 99Відповідь
a.
Neonatal pemphigus
b.
Epidermolysis bullosa
c.
Phlegmon
d.
Vesiculopustulosis
e.
Miliaria
Коментар
Правильна відповідь: Vesiculopustulosis
Питання 100
Відповіді не було
Макс. оцінка до 1,00

Відмітити питання
Текст питання
Two weeks after a case of tonsillitis, a 29-year-old patient noticed facial edema,
weakness, and decreased work capacity. Gradually, he developed shortness of breath, leg
edema, and lumbar edema. Objectively, his skin is pale, his heart sounds are weakened,
he has hydrothorax, anasarca, and blood pressure of 150/100 mm Hg. Clinical urinalysis
shows the following: specific gravity --- 1021, protein --- 9 g/L, erythrocytes 40--50 in
the vision field, hyaline casts --- 4--6 in the vision field. What is the provisional diagnosis
in this case?
Питання 100Відповідь
a.
Acute pyelonephritis
b.
Acute glomerulonephritis
c.
Myxedema
d.
Exacerbation of chronic glomerulonephritis
e.
Heart failure
Коментар
Правильна відповідь: Acute glomerulonephritis
Питання 101
Відповіді не було
Макс. оцінка до 1,00

Відмітити питання
Текст питання
A 45-year-old man, provisionally diagnosed with a transient ischemic attack, was
brought by an ambulance from an aniline-producing factory. Objectively, his skin and
mucosa are cyanotic. His speech is dysarthric. The man is disoriented in space. His blood
test shows the following: erythrocytes -- $4.6 · 10^{12$/L, Нb -- 143 g/L, color index --
0.9, leukocytes -- $5.6 · 10^9$/L, Heinz bodies -- 14%, reticulocytes -- 18%,
methemoglobin -- 36%, ESR -- 5 mm/hour. The patient was diagnosed with a
moderately severe acute aniline intoxication. What antidote agent will be the most
effective in this case?
Питання 101Відповідь
a.
Desferal (Deferoxamine)
b.
Sodium thiosulfate
c.
Pentacin (Calcium trisodium pentetate)
d.
Methylene blue
e.
Succimer
Коментар
Правильна відповідь: Methylene blue
Питання 102
Відповіді не було
Макс. оцінка до 1,00

Відмітити питання
Текст питання
During preventive medical examination of middle and high school students, doctors
were determining, whether the biological development of children matches their
calendar age. For this purpose, the doctors were using the following criteria: annual
increase in body length, ossification of hand bones, number of permanent teeth. What
additional developmental indicator should the doctors include, when examining children
at this age?
Питання 102Відповідь
a.
Development of secondary sexual characteristics
b.
Chest circumference
c.
Vital capacity of the lungs
d.
Body weight
e.
Hand muscle strength
Коментар
Правильна відповідь: Development of secondary sexual characteristics
Питання 103
Відповіді не було
Макс. оцінка до 1,00

Відмітити питання
Текст питання
A patient with epilepsy suddenly developed status epilepticus with generalized seizures
after self-discontinuation of antiepileptic drugs. What are the first-line drugs for the
treatment of this condition?
Питання 103Відповідь
a.
Doxepin, amitriptyline, mianserin
b.
Gabapentin, pregabalin, ethosuximide
c.
Levomepromazine, clozapine, quetiapine
d.
Diazepam, lorazepam, midazolam
e.
Topiramate, oxcarbazepine, carbamazepine
Коментар
Правильна відповідь: Diazepam, lorazepam, midazolam
Питання 104
Відповіді не було
Макс. оцінка до 1,00

Відмітити питання
Текст питання
A 17-year-old girl complains of a pain and swelling of her second finger on the right
hand. Three days ago she made a manicure. The pain developed on the second day after
that. Objectively, her nail fold is swolen, hyperemic, overhangs the nail plate, and is
painful on palpation. What is the most likely diagnosis in this case?
Питання 104Відповідь
a.
Erysipeloid
b.
Subungual panaritium
c.
Cutaneous panaritium
d.
Subcutaneous panaritium
e.
Paronychia
Коментар
Правильна відповідь: Paronychia
Питання 105
Відповіді не було
Макс. оцінка до 1,00

Відмітити питання
Текст питання
Forensic examination of the body of a baby detects the following: weight --- 3500 g,
body length --- 50 cm, the umbilical cord is smooth, moist, shiny, without signs of
drying, lung float tests are positive. What is indicated by the results of the lung float tests
in this case?
Питання 105Відповідь
a.
Secondary atelectasis
b.
Hyaline membrane disease
c.
The child was born alive
d.
The child was born dead
e.
Primary atelectasis
Коментар
Правильна відповідь: The child was born alive
Питання 106
Відповіді не було
Макс. оцінка до 1,00

Відмітити питання
Текст питання
During coke production, the concentration of dust in the air of the working area has
been for many years exceeding the maximum permissible concentration by 4--8 times.
What disease is most likely to develop among the workers in this industry as a result?
Питання 106Відповідь
a.
Silicosis
b.
Anthracosis
c.
Byssinosis
d.
Siderosis
e.
Asbestosis
Коментар
Правильна відповідь: Anthracosis
Питання 107
Відповіді не було
Макс. оцінка до 1,00

Відмітити питання
Текст питання
A 45-year-old man with a history of myocardial infarction developed a brief attack of
palpitations, accompanied by the sensations of lack of air, fear, and vertigo. His blood
pressure is 90/60 mm Hg. ECG during the attack shows extended QRS complex (0.13
seconds) with the heart rate of 160/min., discordant shift of ST segment and T wave,
dissociation of atrial and ventricular rhythm. What disturbance of cardiac rhythm is it?
Питання 107Відповідь
a.
Paroxysmal ventricular tachycardia
b.
Atrial fibrillation
c.
Frequent ventricular extrasystoles
d.
Paroxysmal supraventricular tachycardia
e.
Ventricular fibrillation
Коментар
Правильна відповідь: Paroxysmal ventricular tachycardia
Питання 108
Відповіді не було
Макс. оцінка до 1,00

Відмітити питання
Текст питання
A 6-year-old child became acutely ill and developed fever, headache, and pain during
swallowing. Three hours later, a bright-red fine punctate rash appeared on a hyperemic
skin. The rash is more numerous on the lateral surfaces of the body and in the natural
folds. The oropharyngeal mucosa has a clearly demarcated hyperemia and there is a
purulent plaque on the tonsils. What disease can be suspected in the child?
Питання 108Відповідь
a.
Rubella
b.
Measles
c.
Diphtheria
d.
Infectious mononucleosis
e.
Scarlet fever
Коментар
Правильна відповідь: Scarlet fever
Питання 109
Відповіді не було
Макс. оцінка до 1,00

Відмітити питання
Текст питання
Essential hypertension, as an important non-communicable disease, is the most common
type of arterial hypertension, in which there is a persistent increase in blood pressure up
to 140/90 mm Hg and which is influenced by exogenous and endogenous risk factors.
What factors are endogenous?
Питання 109Відповідь
a.
Obesity
b.
Stress
c.
Excessive emotional and nervous strain
d.
Age (over 40 years), sex, hereditary predisposition
e.
Sensitivity to weather changes
Коментар
Правильна відповідь: Age (over 40 years), sex, hereditary predisposition
Питання 110
Відповіді не було
Макс. оцінка до 1,00

Відмітити питання
Текст питання
A 36-year-old man came to a doctor with complaints of burning retrosternal pain that
usually occurs 1--1.5 hours after eating and becomes worse in a horizontal position.
Esophagogastoduodenoscopy (EGD test) detects two lesion foci in the lower third of the
esophageal mucosa. The foci are up to 5 mm in size and located within one fold. What
tactics would be optimal in this case?
Питання 110Відповідь
a.
Prescription of proton pump inhibitors
b.
Prescription of non-narcotic analgesics
c.
Prescription of clarithromycin
d.
Monitoring in the inpatient department
e.
Surgical treatment
Коментар
Правильна відповідь: Prescription of proton pump inhibitors
Питання 111
Відповіді не було
Макс. оцінка до 1,00

Відмітити питання
Текст питання
A 54-year-old woman came to her family physician for a preventive checkup. Objectively,
the following is observed: height --- 164 cm, weight --- 84 kg, blood pressure --- 130/80
mm Hg. The patient's history indicates the menopause for 5 years. The patient's mother
died of breast cancer, the patient's younger sister suffers from mastopathy. Examination
detects no lumps in the patient's breasts, the organs of her lesser pelvis are within the
age norm. Her cervical cytology results are normal. What advice can be given to this
woman?
Питання 111Відповідь
a.
Undergo an MRI of the breasts
b.
Undergo regular checkups every 3 months
c.
Undergo the next regular checkup in 2 years
d.
Regularly perform breast self-exam
e.
Undergo a mammogram once a year
Коментар
Правильна відповідь: Undergo a mammogram once a year
Питання 112
Відповіді не було
Макс. оцінка до 1,00

Відмітити питання
Текст питання
A patient complains of pain in the lower back, weakness in the right foot, and impaired
walking. Examination reveals painful palpation of lumbar paravertebral points. Cough
impulse sign is positive. The Lasegue sign is positive on the right at the angle of 70°. The
Achilles reflex on the right is absent. There is weakness in the extensor muscles of the
right foot, the patient has problems with standing on the right heel. The patient was
diagnosed with lumbosacral radiculitis and right foot paresis. What group of medicines
must be prescribed in this case?
Питання 112Відповідь
a.
Anticholinesterases
b.
Analgesics
c.
Nonsteroidal anti-inflammatory drugs
d.
Nootropics
e.
Vitamins
Коментар
Правильна відповідь: Nonsteroidal anti-inflammatory drugs
Питання 113
Відповіді не було
Макс. оцінка до 1,00

Відмітити питання
Текст питання
A 14-year-old girl has short stature, broad shoulders, webbed neck, and no signs of
puberty. Her intelligence is normal. Ultrasound of the lesser pelvis shows hypoplasia of
the uterus and the absence of ovaries. Karyotype of the child is 45, X0. What pathological
syndrome can be suspected in this case?
Питання 113Відповідь
a.
Klinefelter syndrome
b.
Down syndrome
c.
Turner syndrome
d.
Edwards syndrome
e.
Patau syndrome
Коментар
Правильна відповідь: Turner syndrome
Питання 114
Відповіді не було
Макс. оцінка до 1,00

Відмітити питання
Текст питання
A 25-year-old pregnant woman complains of fever of 38.5°C that lasts for two days
already, cough, and shortness of breath. She developed these complaints after an
overexposure to cold. Auscultation detects crepitation and localized moist crackles in the
lower part of the right lung. Percussion detects there a dull sound. Complete blood
count shows the following: leukocytes --- $11.0 · 10^9$/L, ESR --- 22 mm/hour. What
antibacterial agent must be prescribed In this case?
Питання 114Відповідь
a.
Amoxicillin
b.
Doxycycline
c.
Levofloxacin
d.
Amikacin
e.
Carbenicillin
Коментар
Правильна відповідь: Amoxicillin
Питання 115
Відповіді не було
Макс. оцінка до 1,00

Відмітити питання
Текст питання
A man, who one day ago returned from a trip to Africa, presents with a sharply painful
cluster of lymph nodes in his armpit. The skin over the lymph node cluster is hyperemic.
Bubonic plague is suspected. What must the contact persons use for urgent prevention
of this disease?
Питання 115Відповідь
a.
Homologous immunoglobulin
b.
Doxycycline
c.
Fluconazole
d.
Heterologous serum
e.
Praziquantel
Коментар
Правильна відповідь: Doxycycline
Питання 116
Відповіді не було
Макс. оцінка до 1,00

Відмітити питання
Текст питання
On day 3 of life, a newborn presented with a deformation, edema, and hematoma of soft
tissues in the left supraclavicular region. The arm is pressed to the torso, passive
movements are accompanied by fussiness of the child. What is the likely diagnosis in this
case?
Питання 116Відповідь
a.
Osteomyelitis of the left clavicle
b.
Displaced fracture of the left clavicle
c.
Erb's palsy
d.
Non-displaced subperiosteal fracture of the left clavicle
e.
Phlegmon of the newborn
Коментар
Правильна відповідь: Displaced fracture of the left clavicle
Питання 117
Відповіді не було
Макс. оцінка до 1,00

Відмітити питання
Текст питання
A 52-year-old woman came to a doctor for her annual preventive checkup. Her father
has a history of cardiovascular disease. Physical examination of the woman shows the
following: height --- 172 cm, weight --- 77 kg, BMI --- 26 kg/m², body temperature ---
36.8°C, pulse --- 81/min., respiratory rate --- 16/min., blood pressure --- 160/100 mm Hg
on both hands. Ophthalmoscopy of the fundus detects narrowing of the retinal vessels.
Laboratory values are within the normal range. What should be prescribed in this case to
reduce the patient's risk of death caused by cardiovascular diseases?
Питання 117Відповідь
a.
Nitroglycerin
b.
Dipyridamole
c.
Warfarin
d.
Enalapril
e.
Aspirin
Коментар
Правильна відповідь: Enalapril
Питання 118
Відповіді не було
Макс. оцінка до 1,00

Відмітити питання
Текст питання
Within 2--3 hours, a 58-year-old man developed multiple spots in his vision, after which
the vision in his right eye darkened. Examination detects eccentric visual acuity of 0.02.
The pupil is moderately dilated, its direct response to light is reduced. Ophthalmoscopy
detects multiple hemorrhages of varying size and shape on the fundus of the eye
("squashed tomato" sign), the optic disc is edematous and hyperemic. The patient has a
history of essential hypertension of II B degree. What is the most likely diagnosis in this
case?
Питання 118Відповідь
a.
Hypertensive angiopathy
b.
Embolism of the central retinal artery
c.
Thrombosis of the central retinal vein
d.
Hypertensive angioneuropathy
e.
Diabetic retinopathy
Коментар
Правильна відповідь: Thrombosis of the central retinal vein
Питання 119
Відповіді не було
Макс. оцінка до 1,00
Відмітити питання
Текст питання
A 48-year-old patient complains of frequent constricting retrosternal pain that radiates
into the left shoulder and left scapula. Pain attacks occur at night, at complete rest, and
last 10--15 minutes. During the visit to a polyclinic, no ECG abnormalities and no
changes in the patient's general condition were detected. What examination would be
most important for clarification of the diagnosis?
Питання 119Відповідь
a.
Echocardiography
b.
Catheterization of cardiac chambers
c.
Bicycle ergometry
d.
Repeat ECG in a week
e.
Holter ECG monitoring
Коментар
Правильна відповідь: Holter ECG monitoring
Питання 120
Відповіді не було
Макс. оцінка до 1,00

Відмітити питання
Текст питання
A 24-year-old woman complains of a papular rash on her external genitalia. The rash is
painless, without itching, clearly separated from the healthy skin. Two months ago, a
round ulcer with a hard smooth bottom located on the patient's labia majora
disappeared on its own without a treatment. What is the likely diagnosis in this case?
Питання 120Відповідь
a.
Measles
b.
Secondary syphilis
c.
Typhus
d.
Toxidermia
e.
Pityriasis versicolor
Коментар
Правильна відповідь: Secondary syphilis
Питання 121
Відповіді не було
Макс. оцінка до 1,00

Відмітити питання
Текст питання
Approximately 40 % of patients with bacterial pneumonia develop concomitant pleural
effusion. This diagnosis can be confirmed by chest X-ray in the direct vertical projection,
if there is at least:
Питання 121Відповідь
a.
200 mL of liquid
b.
300 mL of liquid
c.
100 mL of liquid
d.
500 mL of liquid
e.
---
Коментар
Правильна відповідь: 300 mL of liquid
Питання 122
Відповіді не було
Макс. оцінка до 1,00

Відмітити питання
Текст питання
A 42-year-old patient has been hospitalized into the trauma department. X-ray reveals a
pelvic fracture. Objectively, unassisted urination is not possible, urethrorrhagia is
observed. Palpation detects an enlarged bladder and a painful swelling in the perineum.
What is the provisional diagnosis in this case?
Питання 122Відповідь
a.
Urethral trauma
b.
Reflex urinary retention
c.
Perineal hematoma
d.
Bladder trauma
e.
Acute kidney failure
Коментар
Правильна відповідь: Urethral trauma
Питання 123
Відповіді не було
Макс. оцінка до 1,00
Відмітити питання
Текст питання
A 3-month-old child has been formula-fed since the age of 2.5 months, because the
child's mother has no milk. The mother notes that despite her taking a good care of her
child, the baby developed persistent redness in the skin folds. Three-four days later, the
child developed itching and hyperemic skin patches on the cheeks and chin. The patches
are filled with serous exudate that forms yellowish scabs, when dried out. Seborrheic
scales are observed on the child's scalp. What is the most likely daignosis in this case?
Питання 123Відповідь
a.
Exudative-catarrhal diathesis
b.
Lymphatic-hypoplastic diathesis
c.
Neuro-arthritic diathesis
d.
Staphyloderma
e.
Allergic (atopic) diathesis
Коментар
Правильна відповідь: Exudative-catarrhal diathesis
Питання 124
Відповіді не було
Макс. оцінка до 1,00

Відмітити питання
Текст питання
A baby is 4 days old. The baby's condition after birth is severe (tonic-clonic seizures, no
newborn reflexes). The muscle tone is asymmetrical. The large fontanelle exhibits
increased pulsation and is 3x3 cm in size. The baby's respiration rate is 32/min., with
apnea episodes. The heart sounds are clear and rhytmical, the heart rate is 122/min.
Neurosonogram shows enlarged lateral ventricles with echo-positive inclusions in the
ependyma. What is the most likely cause of the child's condition in this case?
Питання 124Відповідь
a.
Respiratory distress syndrome
b.
Spinal cord birth injury
c.
Meningitis
d.
Hydrocephalus
e.
Intraventricular hemorrhage
Коментар
Правильна відповідь: Intraventricular hemorrhage
Питання 125
Відповіді не було
Макс. оцінка до 1,00

Відмітити питання
Текст питання
For 12 hours, a woodcutter has been working outdoors in windy weather at the air
temperature of +4--5°С. His shoes were compressing the distal parts of his feet. Now he
complains of a pricking and burning pain in his toes 1--3 on both feet, pain in the
interphalangeal joints, and decreased skin sensitivity in toes 1--3. Examination detects
slightly edematous toes, bluish skin, painful and reduced movements in the
interphalangeal joints, slightly reduced skin sensitivity, and retained pulsation of the
arteries on the dorsum of the feet. What is the provisional diagnosis in this case?
Питання 125Відповідь
a.
Critical ischemia of the feet
b.
First degree frostbite
c.
Acute interphalangeal joint arthritis
d.
Obliterating endarteritis
e.
Crush syndrome
Коментар
Правильна відповідь: First degree frostbite
Питання 126
Відповіді не було
Макс. оцінка до 1,00

Відмітити питання
Текст питання
A 35-year-old man was hospitalized with signs of surgical sepsis that has likely been
caused by a large carbuncle in his scapular region. Examination detected secondary
purulent foci in the liver and right lung. What stage of surgical sepsis is it?
Питання 126Відповідь
a.
Purulent resorptive fever
b.
Toxemia
c.
Septicopyemia
d.
Terminal stage
e.
Septicemia
Коментар
Правильна відповідь: Septicopyemia
Питання 127
Відповіді не було
Макс. оцінка до 1,00

Відмітити питання
Текст питання
A 2-year-old child suffers from frequent and long-lasting respiratory diseases and
pancreatogenic malabsorption. Mucoviscidosis (cystic fibrosis) is suspected. What study
is necessary to confirm this diagnosis?
Питання 127Відповідь
a.
Chest X-ray
b.
Karyotyping
c.
Bronchoscopy
d.
Immunogram
e.
Sweat chloride test
Коментар
Правильна відповідь: Sweat chloride test
Питання 128
Відповіді не було
Макс. оцінка до 1,00

Відмітити питання
Текст питання
A 10-year-old boy was treated in the cardiology department for rheumatism, first attack,
active phase, second degree activity. Discharged in a satisfactory condition. What drug
should be prescribed in this case for prevention of secondary rheumatism?
Питання 128Відповідь
a.
Erythromycin
b.
Bicillin-1
c.
Oxacillin
d.
Bicillin-5
e.
Ampicillin
Коментар
Правильна відповідь: Bicillin-5
Питання 129
Відповіді не було
Макс. оцінка до 1,00

Відмітити питання
Текст питання
An 82-year-old woman was hospitalized into the cardiac intensive care unit with
complaints of a sharp pain behind the sternum, a sensation of lack of air, and weakness.
Chest X-ray shows that the transverse size of the cardiac shadow is enlarged, the shape
of the shadow is triangular and has rounded cardiodiaphragmal corners. Cardiac
contractions are of small amplitude and arrhythmic. These X-ray findings most likely
correspond with:
Питання 129Відповідь
a.
Aortic stenosis
b.
Dilated cardiomyopathy
c.
Myocarditis
d.
Exudative pericarditis
e.
Trilogy of Fallot
Коментар
Правильна відповідь: Exudative pericarditis
Питання 130
Відповіді не було
Макс. оцінка до 1,00

Відмітити питання
Текст питання
A 46-year-old patient complains of double vision and drooping of the eyelids that occur
mainly in the second half of the day and almost completely disappear after a rest.
Examination detects slight bilateral ptosis, reduced lateral movement of the eyeballs,
diplopia, and positive proserin (neostigmine) test. Make the diagnosis.
Питання 130Відповідь
a.
Progressive supranuclear ophthalmoplegia
b.
Kearns-Sayre syndrome
c.
Olivopontocerebellar atrophy
d.
Myasthenia
e.
Oculopharyngeal muscular dystrophy
Коментар
Правильна відповідь: Myasthenia
Питання 131
Відповіді не було
Макс. оцінка до 1,00

Відмітити питання
Текст питання
A boy suffering from bronchial asthma started to experience suffocation attacks several
times a day. During the last attack, inhalation of astmopent (orciprenaline) had no effect.
Intensive therapy was unable to stop the attack. The child was transferred into the
intensive care unit with the diagnosis of II degree status asthmaticus. What was the
leading mechanism of the development of this condition in the child?
Питання 131Відповідь
a.
Inflammation of the bronchial mucosa
b.
Complete refractoriness (blockade) of beta-2-adrenoceptors
c.
Increased secretion of bioactive substances by mast cells
d.
Edema of the bronchial mucosa
e.
Spasm of the bronchial smooth muscles
Коментар
Правильна відповідь: Complete refractoriness (blockade) of beta-2-adrenoceptors
Питання 132
Відповіді не було
Макс. оцінка до 1,00

Відмітити питання
Текст питання
A 30-year-old man has received second-degree burns that cover 40% of his body. On
the fourth day after the injury, his general condition acutely deteriorated. He developed
inspiratory dyspnea, frequent cough with frothy sputum, and cyanotic skin. Auscultation
detects numerous wet crackles in the lungs. His blood pressure is 110/60 mm Hg, heart
rate -- 100/min., respiration rate -- 32/min., central venous pressure -- 100 mm H₂O,
total protein -- 50 g/L, Ht -- 30%, Hb -- 90 g/L. ECG shows sinus tachycardia. What
mechanism of pulmonary edema pathogenesis is the main one in this case?
Питання 132Відповідь
a.
Decreased contractility of the myocardium
b.
Pulmonary hypoventilation
c.
Hypervolemia of the pulmonary circulation
d.
Surfactant dysfunction
e.
Decreased plasma osmotic pressure
Коментар
Правильна відповідь: Decreased plasma osmotic pressure
Питання 133
Відповіді не було
Макс. оцінка до 1,00

Відмітити питання
Текст питання
The condition of a full-term newborn deteriorated on the first day of life. The baby was
born from the third pregnancy, during the second half of which gestosis was observed.
The mother's blood group is 0(I) Rh(-). Examination shows that the baby is inert and has
icteric skin and mucosa; baby's urine and stool are of normal color. Blood serum bilirubin
is 248 mcmol/L, because of indirect bilirubin levels. What is the most likely cause of this
pathologic condition?
Питання 133Відповідь
a.
ABO incompatibility
b.
Physiological jaundice
c.
Rh incompatibility
d.
Fetal hepatitis
e.
Biliary atresia
Коментар
Правильна відповідь: Rh incompatibility
Питання 134
Відповіді не було
Макс. оцінка до 1,00

Відмітити питання
Текст питання
After giving birth, a 25-year-old woman developed increased weakness in her legs and
unsteady walking. She has been suffering from this condition for 6 years already. Every
autumn, she notes a deterioration of her condition. Objectively, the woman is euphoric
and exhibits a reduced critical attitude towards her condition. She has horizontal
nystagmus, high tendon reflexes, foot clonus, pathological foot reflexes, no abdominal
reflexes, ataxia during the Romberg test, and intentional tremor and missing during
coordination tests. Temporal pallor of the optic discs is observed on the eye fundus.
What is the most likely diagnosis in this case?
Питання 134Відповідь
a.
Multiple sclerosis
b.
Acute disseminated encephalomyelitis
c.
Dyscirculatory encephalopathy
d.
Myasthenia gravis
e.
Amyotrophic lateral sclerosis
Коментар
Правильна відповідь: Multiple sclerosis
Питання 135
Відповіді не було
Макс. оцінка до 1,00

Відмітити питання
Текст питання
A 59-year-old woman complains of pain and edema in the small joints of her hands,
shortness of breath, weakness. This condition lasts for 8 years already. Objectively, her
body temperature is $37.8^oC$, she has fine punctate hemorrhages on her torso and
limbs, ulnar deviation of the hands is observed. The borders of the heart are shifted to
the left, a systolic murmur can be detected over the apex. Her pulse is 96/min. Her blood
pressure is 170/100 mm Hg. Complete blood count shows the following: erythrocytes ---
$3.2 · 10^{12$/L, Hb --- 108 g/L, leukocytes --- $6.8 · 10^9$/L, platelets --- $220 ·
10^9$/L, ESR --- 48 mm/hour, C-reactive protein (+++). General urinalysis shows the
following: specific gravity --- 1016, protein --- 2.8 g/L, leukocytes --- 10--12 in the vision
field, erythrocytes --- 2--4 in the vision field. What is the most likely diagnosis in this
case?
Питання 135Відповідь
a.
Rheumatoid arthritis
b.
Rheumatism
c.
Thrombocytopenic purpura
d.
Chronic glomerulonephritis
e.
Systemic lupus erythematosus
Коментар
Правильна відповідь: Rheumatoid arthritis
Питання 136
Відповіді не було
Макс. оцінка до 1,00
Відмітити питання
Текст питання
A 10-year-old boy came to the polyclinic with complaints of stuffy nose. It is known that
these signs occur in the child periodically (in spring and autumn). He has a history of
atopic dermatitis. The father of the child has bronchial asthma. Objectively, the boy's face
is pale and slightly swollen. Respirations are 22/min. Auscultation detects vesicular
respiration over the lungs. Rhinoscopy shows swollen and pale nasal mucosa. What
disease can be suspected?
Питання 136Відповідь
a.
Acute maxillary sinusitis
b.
Recurrent respiratory disease
c.
Acute adenoiditis
d.
Acute rhinitis
e.
Allergic rhinitis
Коментар
Правильна відповідь: Allergic rhinitis
Питання 137
Відповіді не було
Макс. оцінка до 1,00

Відмітити питання
Текст питання
A 5-year-old child became acutely ill with the fever of 39.2°C, one episode of vomiting,
complaints of cramping pain in the abdomen, tenesmus, and frequent bowel movements
that produce a small amount of feces and a large amount of mucus with pus and blood
streaks. Examination detects a dense sigmoid colon that is painful to palpation. Make the
diagnosis.
Питання 137Відповідь
a.
Shigellosis
b.
Rotavirus infection
c.
Cholera
d.
Salmonellosis
e.
Acute appendicitis
Коментар
Правильна відповідь: Shigellosis
Питання 138
Відповіді не було
Макс. оцінка до 1,00

Відмітити питання
Текст питання
A woman with polycystic kidney disease observes an increase in her 24-hour urine
output to 2--2.5 liters. Dynamic nephroscintigraphy shows the following: total
glomerular filtration rate --- 34 mL/min., serum creatinine --- 84 mcmol/L, urea --- 8.0
mmol/L. What stage of chronic kidney failure is it?
Питання 138Відповідь
a.
Terminal
b.
Latent
c.
Compensated
d.
Intermittent
e.
Polyuric
Коментар
Правильна відповідь: Compensated
Питання 139
Відповіді не було
Макс. оцінка до 1,00

Відмітити питання
Текст питання
A person has been hospitalized with the diagnosis of trichinellosis. What food product is
the likely cause of this helminthiasis?
Питання 139Відповідь
a.
Mutton
b.
Poultry
c.
Beef
d.
Rabbit
e.
Pork
Коментар
Правильна відповідь: Pork
Питання 140
Відповіді не було
Макс. оцінка до 1,00

Відмітити питання
Текст питання
A 30-year-old woman complains of menstruations missing for a year. She has a history
of a massive postpartum hemorrhage. Objectively, her secondary sex organs are
normally developed, the hair growth pattern is of the female type. Bimanual examination
detects normal uterus and uterine appendages. What is a possible cause of amenorrhea
in this case?
Питання 140Відповідь
a.
Adrenocortical necrosis
b.
Pituitary necrosis
c.
Ovarian insufficiency
d.
Thyroid disorder
e.
Hypothalamic insufficiency
Коментар
Правильна відповідь: Pituitary necrosis
Питання 141
Відповіді не було
Макс. оцінка до 1,00

Відмітити питання
Текст питання
hours after eating unknown mushrooms, a 28-year-old man sensed a decrease in his
mobility and deterioration of his ability to focus. This condition was then followed by a
state of agitation and agression. On examiantion he is disoriented and his speech is
illegible. 4 hours later he developed fetor hepaticus and lost his consciousness. What
syndrome can be observed in this patient?
Питання 141Виберіть одну відповідь:
Acute hepatic failure
Hepatolienal syndrome
Portal hypertension
Cholestatic syndrome
Cytolytic syndrome
Коментар
Правильна відповідь: Acute hepatic failure
Питання 142
Відповіді не було
Макс. оцінка до 1,00

Відмітити питання
Текст питання
A 75-year-old man in a severe condition suffers from dyspnea at rest, marked weakness,
and arrhythmia. Abdominal aortic pulsation is observed, further on there is a systolic
murmur detected. Palpation reveals a volumetric formation in the mesogastrium. Blood
pressure is 70/40 mm Hg. There is no pulsation over the femoral arteries. Oliguria is
detected. Which diagnosis is the correct one?
Питання 142Відповідь
a.
Cardiosclerotic aneurysm
b.
Acute pericarditis
c.
Dissecting aortic aneurysm
d.
Acute cardiac aneurysm
e.
Pancreatic cyst
Коментар
Правильна відповідь: Dissecting aortic aneurysm
Питання 143
Відповіді не було
Макс. оцінка до 1,00

Відмітити питання
Текст питання
A premature newborn boy (pregnancy 5, birth 1, gestation term --- 27 weeks) has
irregular respiration of <30/min. after birth and $SpO_2$ of 70 %. Retractions of the
pliable areas of the chest and expiratory groaning are observed. Without oxygen
support, generalized cyanosis occurs. Auscultation reveals crepitus in the basal
segments. There are no data on the prevention of respiratory distress syndrome in the
baby. What treatment tactics must be chosen in this case?
Питання 143Відповідь
a.
Administer the surfactant preparation intratracheally within the first 15 minutes after
birth
b.
Provide oxygen therapy using an oxygen tent
c.
Administer the surfactant preparation intratracheally 2 hours after birth
d.
Provide artificial pulmonary ventilation with an Ambu bag and a mask
e.
Prescribe an antibacterial therapy
Коментар
Правильна відповідь: Administer the surfactant preparation intratracheally within the
first 15 minutes after birth
Питання 144
Відповіді не було
Макс. оцінка до 1,00

Відмітити питання
Текст питання
A 65-year-old patient was prescribed pharmacotherapy (ramipril, atorvastatin,
amlodipine, acetylsalicylic acid, bisoprolol) for essential hypertension with concomitant
ischemic heart disease. Two weeks later, he consulted a doctor about a dry cough.
Examination detects no signs of acute respiratory viral infection or damage to the
bronchopulmonary apparatus. A side effect of ramipril is suspected. What drug can be
used to replace ramipril in the treatment plan?
Питання 144Відповідь
a.
Nifedipine
b.
Torasemide
c.
Valsartan
d.
Nebivolol
e.
Enalapril
Коментар
Правильна відповідь: Valsartan
Питання 145
Відповіді не було
Макс. оцінка до 1,00

Відмітити питання
Текст питання
Among the population living near a pesticide production factory, the number of
congenital malformations that manifest as central paralysis, idiocy, and blindness of
newborns is dynamically increasing. Compounds of what chemical substance can cause
the development of these pathologies?
Питання 145Відповідь
a.
Iron
b.
Chrome
c.
Mercury
d.
Strontium
e.
Cadmium
Коментар
Правильна відповідь: Mercury
Питання 146
Відповіді не було
Макс. оцінка до 1,00

Відмітити питання
Текст питання
A 9-year-old boy is in a severe condition. His body temperature is 38--39°C, he has
nosebleeds and complains of pain in his bones. Objectively, the boy presents with acute
pallor, hemorrhagic rash, and ulcerative necrotizing stomatitis. All the groups of lymph
nodes are enlarged. The liver is +5 cm. The spleen is +4 cm. What test will be decisive for
diagnosis-making in this case?
Питання 146Відповідь
a.
Complete blood count
b.
Abdominal ultrasound
c.
Immune complex testing
d.
Myelogram
e.
X-ray of the mediastinum
Коментар
Правильна відповідь: Myelogram
Питання 147
Відповіді не було
Макс. оцінка до 1,00

Відмітити питання
Текст питання
The mother of a 6-month-old boy came to a pediatrician. She is concerned that her child
has not received any vaccinations. There are no health complaints. Objectively, the
following is observed: body temperature --- 37.1°C, respiratory rate --- 20/min., blood
pressure --- 100/70 mm Hg, the child's height and weight are within the range between
the 50th and 75th percentile. What tactics must the doctor choose in the immunization
of this child according to the regulatory documents issued by the Ministry of Health?
Питання 147Відповідь
a.
Administer BCG, MMR, and DPT vaccines
b.
Postpone the vaccination and make an appointment in a week
c.
The child requires no vaccination
d.
Administer BCG vaccine
e.
Perform a Mantoux test and make an appointment in 2 days
Коментар
Правильна відповідь: Perform a Mantoux test and make an appointment in 2 days
Питання 148
Відповіді не було
Макс. оцінка до 1,00

Відмітити питання
Текст питання
A 38-year-old pregnant woman, gravida 2, gestation term of 23--24 weeks, has type 2
diabetes mellitus, for which she receives metformin in the dose of 2500 mg per 24 hours.
Laboratory analysis detects glycated hemoglobin (HbA₁c) levels of 7.2 %. What further
treatment strategy should be chosen for this patient?
Питання 148Відповідь
a.
Continue the treatment unchanged
b.
Add GLP-1 analogues to metformin
c.
Increase the dose of metformin to 3000 mg per 24 hours
d.
Discontinue metformin and switch to sulfonylureas
e.
Prescribe insulin therapy and discontinue metformin
Коментар
Правильна відповідь: Prescribe insulin therapy and discontinue metformin
Питання 149
Відповіді не було
Макс. оцінка до 1,00

Відмітити питання
Текст питання
An 18-year-old adolescent bought at a store a salad with smoked fish, mushrooms, and
mayonnaise. Six hours after eating the salad, he developed progressive weakness, vision
impairment, "fog in the eyes", and problems with swallowing. He was hospitalized.
Examination detects the body temperature of 36.0°C and pale skin and mucosa. The
patient is adynamic, answers questions sluggishly. The pupils are dilated, with reduced
response to light. The patient's voice is hoarse, his oral mucosa is dry. What type of food
poisoning can be suspected in this case?
Питання 149Відповідь
a.
Staphylococcal intoxication
b.
Fish poisoning
c.
Botulism
d.
Mushroom poisoning
e.
Salmonellosis
Коментар
Правильна відповідь: Botulism
Питання 150
Правильно
Балів 1,00 з 1,00

Відмітити питання
Текст питання
A 73-year-old man has a 5-year-long history of benign prostatic hyperplasia. One
morning he developed an acute urinary retention. Catheterization of his bladder is
impossible. What urgent aid must be provided for this man?
Питання 150Відповідь
a.
Suprapubic bladder tap
b.
Diuretics
c.
Thermal procedures
d.
Antispasmodics and analgesics
e.
Adrenergic blockers
Коментар
Правильна відповідь: Suprapubic bladder tap
Завершити перегляд

You might also like